You are on page 1of 115

Chapter 1 Functions, Limits, and Continuity

Term 1 Week 1 and 2

Section 1.2 Limit of a Function

Basic

1. How close to 8 must x be for the values of h(x) = 4x + 1 to be within 0.2 from 33?

2. Evaluate the following limits.


1
a) lim b) lim  tan x 
x 1 x  3 x

3. Evaluate the following limits.


 
sin 2  x  
 2 x2  2 x  1 x3  8
a) lim b) lim c) lim

x    
2 x 1 x2 1 x2 x2  4
2  x  2 
2

  

4. Find:
sin(sin x) cos x  1
a) lim b) lim
x 0 x x  0 sin x

5. Given lim  2 x  1  5 . Find the value of  that is guaranteed by the definition of


x2

the limit to exist and that corresponds to   0.

6. Find the limit in each case. If the limit does not exist, state so.
{[] is the greatest integer function.}
4x 4x 4x
a) lim b) lim c) lim
x 2 x  2 x 2 x  2 x 2 x  2

4  x2 4  x2 4  x2
d) lim e) lim f) lim
x 2 x2 x 2 x2 x 2 x  2

Level N | 1
Calculus I

Core Grid

1. Find the limit in each case.


If the limit does not exist, enter 000 for your answer.
If the limit is infinite, enter 999 for your answer.
3x 2  x  2 2 x 3  4 x  12
a) lim 2 b) lim
x  x  4 x  1 x  3x 2  x
1  5 x  x5 6  5x2
c) lim d) lim 2
x  2 x3 x  3 x  7

x6  3x 4  1 24 x 2  x  12
e) lim 7 f) lim 3
x  x  7 x 3  1 x  8 x  3 x  11

x  256
2
2. What whole number is equal to lim ?
x 16 x 4
3. Find the limit in each case.
If the limit does not exist, enter 000 for your answer.
If the limit is infinite, enter 999 for your answer.
2  2cos x sin x
a) lim b) lim
x 0 sin 2 x x 0 cos x 1  cos x 
cos x 2
c) lim d) lim
x 0 1  cos x x 0 1  cos x

5sin x  sin10 x tan18 x


e) lim f) lim
x 0 3x x 0 2x

4. A function f defined by f (x) = 5x  6 is continuous at x = 0 because for every  > 0,


there exists  > 0 such that | f (x) + 6 | <  whenever | x | < .
The largest value of  when  = 0.1 is 1/k, where k is a whole number.
What is the value of k?
k  x if x  3
5. Consider the function f  x    2 .
4 x if x  3
For what value of k does lim f ( x) exist?
x 3

6. Below is the graph of a function f. Use this graph to answer the following question.
f(x)
8

4
x
0

Level N | 2
Calculus I

Find the limit in each case.


If the limit does not exist, enter 000 for your answer.
If the limit is infinite, enter 999 for your answer.
a) lim f ( x) b) lim f ( x) c) lim f ( x )
x 0 x 0 x 0

x5
7. Find lim .
x 5 x  6 1
If the limit does not exist, enter 000 for your answer.
If the limit is infinite, enter 999 for your answer.
8.
f(x)

x
0 1 2 3
2
The function f is defined for x ≥ 1 by the graph above.
The function g is defined for all x < 1 and such that lim g ( x) exists.
x 1

 g  x  if x  1

The function h is defined over the set of real numbers h  x    .
 f  x  if x  1

What must the value of lim g ( x) be If lim h( x) exists?
x 1 x 1

Level N | 3
Calculus I

Section 1.3 Continuity

Basic

1. Determine whether the function is continuous or not at the indicated point. In the
event the function has a removable discontinuity, extend the definition of the
function so that the new function is continuous at the given point.
1  x2 x2  2x
a) f ( x)  2 ; x  0 b) f ( x)  ;x  0
x x

2. Study the continuity of the function f on the interval (0, 1) in each of the following
cases:
 2 x if x  0.5  x  1 if x  0.5
a) f ( x)   b) f ( x)  
1 if x  0.5 1 if x  0.5

3. Show that the function c given by c( x)  csc x is continuous on any open interval
that does not contain any of the values x = k, k  Z.

4. Consider the function whose graph is shown below.


y
3
2
1
x
3 2 1 0 1 2 3
1
2
3
Determine which of the following statements are true and which are false?
a) f is left continuous at x = 1.
b) f is right continuous at x = 1.
c) f is continuous at x = 1.
d) f is continuous on (3, 1).
e) f is continuous on [3, 1].
f) f is continuous on [1, 1].
 x  1 if x  0

5. Consider the function g given by g ( x)   x  1 if 0  x  1
5 x if x  1

Determine, whether g is continuous on (0, 1]. Explain.

6. Show that the equation x3  3x  0 has a solution between x = 1 and x = 2.

Level N | 4
Calculus I

Core Grid
 x  1 if x  2

1. Consider the function f  x    x  2 if  2  x  2 .
2 x if x  2

Which of the following statements are true?
Select all that apply and enter the labels of the correct statements in an ascending
order without spaces or separators.
lim f  x   f (2)
1. x2
lim f  x   f (2)
2. x2
3. f (x) is right continuous at x = 2.
4. f (x) is left continuous at x = 2.
5. f (x) is continuous at x = 2.
6. f (x) is discontinuous at x = 2.

2. Consider the function f(x) = |x  3|.


Which of the following statements are true?
Select all that apply and enter the labels of the correct statements in an ascending
order without spaces or separators.
1. f(3) exists.
2. lim f  x  exists.
x 3
3. lim f  x  does not exist.
x 3
4. lim f  x  exists.
x 3
5. f(x) is continuous at x = 3.
6. f(x) has a removable discontinuity at x = 3.

3x if x  2
3. Consider the function f  x    .
2 x  2 if x  2
Which of the following statements are true?
Select all that apply and enter the labels of the correct statements in an ascending
order without spaces or separators.
1. f(2) exists.
2. f(x) is not continuous at x = 2 because it has a gap.
3. f(x) is continuous at x = 2 because f(2) exists.
4. f(x) is not continuous at x = 2 because it has a jump.
5. lim f  x  exists.
x 2
6. f(x) is continuous at x = 2 because f  2   lim f  x  .
x 2

Level N | 5
Calculus I

 x 2  36
 when x  6
4. Consider the function f ( x)   x  6 .
m when x  6

For what value of m is the function f continuous over the set of real numbers?
Enter your answer as a whole number.

 sin12 x
 when x  0
5. Consider the function g ( x)   3x .
b when x  0
What is the value of b that makes g(x) continuous over the set of real numbers?
Enter your answer as a whole number.

6. How many points of discontinuity does the function


 3x
 2 x  4 when x  1
f ( x)   have?
 6 x when x  1
 x  3

7. The figure below shows the graph of f(x). Use this graph to answer the following
question.
f (x)
8

x
–2 2 4

How many points of discontinuity does the function f have?

Level N | 6
Calculus I

AP Grid

3sin x  4  12 if x  4
2

1. Consider the function f ( x)   2 .



 kx  148 if x  4
For what value of k is the function f continuous at x = 4?

2. The function f is defined and continuous for all values of x  0.


x 2  8x  9
Moreover, f (x )  for x  9 and f (9) = k.
6 x  18
What is the value of k?

Level N | 7
Calculus I

Chapter 2 The Derivative


Term 1 Week 3

Section 2.1 Rates of Change


AP Grid

200x
1. What is the average rate of change of the function g  x   over the interval
x 1
[1, 3]?

2. What is the value of c if ky  5x = c is tangent to the graph of f (x )  x 2  11 at


(5, 6)?

3. Given a differentiable function f such that f (2) = 21.


The graph of f , the derivative of f, for 4  x  4 is shown below.
6 y
Graph of f 
4

2
x
6 4 2 0 2 4 6
(4, ) 2

4

6

What is the value of c if (1, c) is a point on the tangent to the graph of f at x = 2?
[Hint: Find the equation of the tangent and substitute 1 for x.]

4. The table below shows selected values of C(t) for different values of t, 5  t  47. It
is also given that C is a differentiable function of t.

t 5 11 17 23 29 35 41 47
C(t) -4 8 56 83 27 13 7 0

Based on the values in the table, what is the best approximation to C(14)?

Level N | 8
Calculus I

5. Consider the data given in the following table.

t 10 12 13 14 17 18 20 21
L(t) 12 35 92 78 110 134 144 90

Based on the values in the table, what is the average rate of change of L(t) over the
interval [14, 20]?

Section 2.2 Definition of the Derivative

Basic

1. Given f (x) = x2. Find f (x) using the definition of the derivative and deduce the
value of f (4).

, find g   x  using the definition of the derivative. Deduce the


1
2. Given g  x  
x2
value of g  1 . Give the value(s) of x, for which g and g  are not defined.

3. Show that the function g given by g(x) = x | x | is differentiable at x = 0.

3
4. Show that the function n given by n(x) = x has a vertical tangent at x = 0.

5. Show that the function f given by f (x) = | x  3| is not differentiable at x = 3.

Core Grid

ax3  bx  11 x  1
1. Given that the function f ( x )   is differentiable at x = 1,
 2 x  1 x  1
what is the value of f(–2)?
Enter your answer as a whole number.

dy π
2. Let y  3x cot x . Find the value of when x  , and enter the answer rounded
dx 4
to the nearest whole number.

4 x  1 if x  0
3. Consider the function f  x    2
3 x if x  0 .
Which of the following statements are true?

Level N | 9
Calculus I

Select all that apply and enter the labels of the correct statements in an ascending
order without spaces or separators.
1. f(x) is not differentiable at x = 0 because f(0) does not exist.
2. f(x) is not continuous but is differentiable at x = 0.
3. f(x) is not differentiable at x = 0 because it is not continuous at that point.
4. f(x) is differentiable at x = 0 because f(0) exists.
f  h   f  0
5. lim does not exist.
h 0 h
f  h   f  0
6. lim exists.
h 0 h

dy π
4. If y = 3cot(  6x), what is the value of when x = ?
dx 24
Enter the answer as a whole number.

2 x  1 if x  0
5. Consider the function f  x    .
 x + 1 if x  0
2

Which of the following statements are true?


Select all that apply and enter the labels of the correct statements in an ascending
order without spaces or separators.
1. f (0) exists.
2. lim f ( x) does not exist.
x 0

3. lim f (h)  f (0) does not exist.


h 0 h
4. f(x) is continuous at x = 0.
5. f(x) is differentiable at x = 0.

Section 2.3 Rules for Finding Derivatives

Basic

1. Given y, find y .
a) y = x3 + x2 b) y = x(x2 + x  1)

2. Given y, find y .
1 3 2 4
a) y = 8x3  5x2  3x b) y  x  x
4 3

3. Find the point(s) on the curve y  x3  12x  5 where the normal(s) is (are) vertical.

4. Given y = (x3  8)(x2 + 1), find y x1 .

Level N | 10
Calculus I

x
5. Given y  , find y .
x  2x 1
3

6. Given y, find y  and y .


1
a) y = 4x3  x  2 b) y  x 
x

7. Given y   4 x3  x 2  1 , find y .
3

AP Grid

1. The twice-differentiable function f is defined for all real numbers and satisfies the
following conditions: f  0 = 2, f   0  = 3, and f   0  = 0.
Find the value of g   0 if the function g is given by g(x) = x f 3  x  .

2. The differentiable function f is defined for all real numbers and satisfies f (1) = 8
and f '(1) = 2. The function g is given by g(x) = f 3  x3  6 x  5 for all real
1
3
numbers. What is the value of g   2  ?

Section 2.4 The Derivative of Trigonometric Functions

Basic

1. Given y  x sin (x3  8), find y .

2. Given y  x2 cos (x2  x  1), find y

Term 1 Week 4

x sec x dy
3. Given y  , find .
x 1 dx

AP Grid

3 1
1. What is the rate of change of the function h  x   tan  3x  at x  ?
 12

Level N | 11
Calculus I

Chapter 3 Graphs of Functions

Section 3.1 Extreme Values and the Mean Value Theorem

Basic
1. Find the absolute maximum and the absolute minimum for f ( x)  x where
x   2,1 .

2. Given s( x)  sin x . State why there must be at least one point c in (0, 2π) for which
s  c   0 . Find all such values.

f (3)  f (2)
3. Find the value(s) of c that satisfies f (c)  for the
3  (2)
x 1
function f ( x)  .
x 1

Core Grid
1. Consider the function f ( x)  3x3  9 x2 , where x [2,1] . Let M be the absolute
maximum of the function, and let m be the absolute minimum of the function.
What whole number is equal to the difference M – m?

x
2. Consider the function f ( x)  4sin over the interval [0, k]. What is the smallest
4
positive value of k for which the function has at least one point c in the given
interval where f (c) = 0? Enter your answer as a whole number.

3. Consider the function f(x) = 2x2 + 14x over the interval [0, k]. What is the value of
k for which the function has at least one point c in the given interval where
f (c) = 0? Enter your answer as a whole number.

x
4. Consider the function f  x   2 tan .
2
Which of the following statements are true?
Select all that apply and enter the labels of the correct statements in an ascending
order without spaces or separators.
1. f(x) is continuous over the interval  0, π  .
2. f(x) is differentiable over the interval  0, π  .
3. f(x) is differentiable at x = 0.
4. f(x) is continuous over the interval  0, π  .
f     f (0)
5. There exists at least one value c   0,   such that f (c)  .

Level N | 12
Calculus I

AP Grid

1. Let f be a twice-differentiable function such that

f (3) = 1, f (3) = 4, and f (1) = 11.

f (4) = 6, f (4) = 12, and f (6) = 13.

Let g be the function given by g(x) = f (f (x)).


What number must be used to fill in the blank?
By the Mean Value Theorem, there must be a value k in the interval (3, 4) such
that g(k) is equal to ______.

2. The table below shows selected values of a differentiable function y(x).

x 0 5 10 15 20 25
y 5 20 44 88 115 123

The Mean Value Theorem ensures the existence of a value of x in the interval [0, k]
for which y  = 6. Based on the values given in the table, what is a smallest value for
k?

Level N | 13
Calculus I

Term 1 Week 5

Section 3.2 Sense of Variation

Basic

x2
1. Find all asymptotes of the function y  .
x2  x  2

2. Find all critical values of f ( x)  x  2  x  and all intervals where the values of f (x)
3

are increasing and those where the values of f (x) are decreasing.
Deduce whether a critical point corresponds to a local maximum, a local minimum,
or a saddle point.

3. Study the behavior of f ( x)  5 x 4  7 x3  12 x 2  8 as x   and as x   .

4. Consider the function f given by y  f ( x)  2 x3  3x 2  36 x  1 .


a. Using the second derivative test, identify each of its extreme values as a local
maximum or a local minimum.
b. Find the intervals over which the curve y = f (x) is concave up and those where it
is concave down. Identify the inflection points, if any.

AP Grid

1. Consider a twice-differentiable function f over the set of positive real numbers. The
derivative f  of f is given by f ′(x) = (21 – x) x−4. What is the largest value of k for
which f is concave down on the interval (0, k)?

64
2. The function y  f ( x)  4 x  has local extreme points. What is the value of y at
x
the local minimum?

3. Let f be the function defined by y = f (x) = 6x3 + ax2 + 4x  21 and having a point
of inflection at x = 1. What is the value of a?

4. The graph of the piecewise-linear function g, the derivative of g, over the interval
(6, 6) is shown below.

Level N | 14
Calculus I

6 y

2
x
6 4 2 0 2 4 6
2

4

6
Fill in the blanks in what follows and give a justification for each.
The graph of y = g(x) has a point of inflection at x = ____.
g has a local maximum at x = ____.
g has a local minimum at x = ____.

Level N | 15
Calculus I

Section 3.3 Curve Sketching

Trigonometric graphs from the PC (exercise book)


Basic

1. Sketch the curve y  f ( x)  3x 4  4 x3 .

x2  x  3
2. Sketch the graph of y  .
x2

3. Study the variation and draw the graph of y  f ( x)  x  x 2  1 .

4. Study the variation and draw the graph of the function given by y = sin x + cos 2x.

yx  yx 2  3t 2  4t
5. a. Verify the rule y ''  for x  ,y . (The answer should be
 x 1 t 1 t
3

zero as y '  2 .)
2  3t 2  4t
b. By eliminating t from x  ,y , show that the equation becomes
1 t 1 t
y  2x  2, x  3 .

Core Grid

1. Consider the function given parametrically by x  t , y  t 2 , t  0 .


d2y
 kt , where k is a whole number to be determined. What is the value of k?
dx 2

2. Consider the three functions given parametrically by


f : x  0.75t , y  t g: x = t2, y = t9 h: x = t5, y = t

Consider also the five graphs below labelled I, II, III, IV, and V.
Select all that apply and enter their labels in the same order as they appear
(ascending order).
Enter the labels without any spaces or commas.
1. The graph of f is the curve labelled I.
2. The graph of f is the curve labelled II.
3. The graph of h is the curve labelled V.
4. The graph of h is the curve labelled IV.
5. The graph of g is the curve labelled IV.

Level N | 16
Calculus I

6. The graph of g is the curve labelled III.


y
y I y II III

x
O

x x
O O
y y
IV V

x x
O O

Parametric graphs what is in the PC(exercise book)

Level N | 17
Calculus I

Chapter 4 Applications of the Derivative

Section 4.1 Rate of Change


AP Grid

1. Two particles move along the x-axis. For 0 ≤ t ≤ 35, the position of particle P at
1
time t is given by p(t) = 3 while the position of particle R at time t is given by
t 1
r (t )  2t 2  2t  3 . The two particles travel in opposite directions over the time
interval a < t < b. What is the maximum value of b?

2. A particle moves along the x-axis so that its position x at time t  0 is given by
x(t )  t  12 t  23
For what value of t, in the open interval 0 < t < 225, does the particle change
direction?

3. A particle moves along the x-axis. The position of the particle at time t  0 is given
by p(t )  2t 3  102t 2  40 . At what time does the particle start slowing down for the
first time?

4. A particle starts moving along the x-axis at time t = 0. The position of the particle at
time t  0 is given by p(t )  t 3  33t 2  12 . At what time does the particle start
moving to the right?

5. A particle moves along the x-axis so that its position x at time t  0 is given by
x  t   7  t 3  4t  1 . What is the velocity of the particle at t  1 ?
4

6. A particle moves on the x-axis. Its position at any time t  0 is given by


2
x  t    t 3  6t 2  12t  18 . At what position does the particle attain its maximum
3
velocity for the first time?

7. The velocity time graph of a particle moving on a straight line is shown below.

Level N | 18
Calculus I

v, in m/s
6

4 (40, 4)

0 45
10 33 t, in s
2 (25, 2)
(30, 2)
4
(17, 4)
What is the length of the longest time interval over which the particle is
continuously moving in the negative sense (negative velocity)?

8. The velocity-time graph of a particle moving on a straight line is shown below. At


t = 0, the position of the particle is x(0) = 12.
v(t)
4
3

1
t
0
1 2 3 4 5 6 7 8 9 10
1

2

1 2 c
The position of the particle at any time t for 3 ≤ t ≤ 5 is given by x(t )   t  5t  .
2 2
What is the value c?

Level N | 19
Calculus I

9. The velocity-time graph of a particle moving along a straight line is shown below.
v(t)
40
30

20

10
t
0
1 2 3 4 5 6 7 8 9 10
10

20

What is the average acceleration of the particle in the time interval 1 ≤ t ≤ 9?


Express your answer as a decimal.

10. A particle moves along the x-axis. The signs of its velocity v and its acceleration a
change according to the chart below.
t 0 1.5 3.0 4.5 6.0 7.5 9.0

v(t) + +   + +

a(t)    + + +
At what time does the speed begin to increase for the first time?

11. The velocity time graph of a particle moving on a straight line is shown below.
v(t)
60
45

30

15
t
0
1 2 3 4 5 6 7 8 9 10
15

30

What is the acceleration of the particle when it is speeding up at the fastest rate?

Level N | 20
Calculus I

Section 4.2 Related Variables

Basic

1. Given 2 xy  y 2  x  1 .
dy d2y
a. Find . b. Find .
dx dx 2

Term 1 Week 6
d2y π 
2. Given y  sin  xy  , find at the point  , 0  .
2 
2
dx

3. Find equations for the tangent and normal to the curve given by x  y  y at the
point  2, 2 .

Core Grid

1. Consider the implicit function y2  x2 = 64. Derive a simplified expression for


d2y
and find its value when y = 2. Enter this value as a whole number.
dx 2

Level N | 21
Calculus I

AP Grid NUMBERS 11, 2, 4

Section 4.4 Linearization

Basic

1
1. Find the linearization of f ( x)  at x = 0. Use the linear model of f to obtain
x2  1
1
an estimate for .
1.04

Core Grid

1. Consider the function f given by y  f ( x)  x3  3x 2  2 x . Let L(x) be the


linearization of f at x = 0. What is the value of L(10)?

Level N | 22
Calculus I

Section 4.5 Other Numerical Methods for Solving Equations

Basic

1. Show that the equation x  e x  0 has a root between 0.4 and 0.6. Use the
iterative formula xn 1  xne  xn to find the root to one decimal place.

Chapter 5 Integrals

Term 1 Week 7

Section 5.1 The Indefinite Integral

Basic

  2 x  5  4 csc x  dx .
2
1. Integrate:

2. Integrate:  cos 2 x dx .

3. Given y '  sin  2 x  and y = 4 when x = 0, find y.

4. Integrate.
 1  5
a)  dx b)  dy
  2 x  1 y2
4
 3

 cos x sin 3 x
d) 
1 1 1
c)  dx  2 sin cos d
 x   

Core Grid

 36 x 2 
1. If   3  15sin 3x  dx  I ( x)  C , find the value of I  π  .
  
Enter your answer as a whole number.

2. If (192 x 2  8  3π sec2 πx) dx  I ( x)  C , find the value of I   .


1
 4
Enter your answer as a whole number.

Level N | 23
Calculus I

4
3. If (12cos x  6sin x)dx  I ( x)  C , find the value of I ( ) , where cos =
 5
and
3
sin = .
5
Enter your answer as a whole number.

2 x  7 x2 1
4. If x 4 dx  I ( x)  C , find the value of I  3  .
Enter your answer as a whole number.

x x3  9 dx  I ( x)  C , find the value of I (3) .


2
5. If
Enter your answer as a whole number.

π

6. If 21tan 2 x sec 2 x dx  I ( x)  C , find the value of I   .
4
Enter your answer as a whole number.

2
7. If 3x  25
dx  I ( x)  C , find the value of I (100) .

Enter your answer as a whole number.


8. If 320cos x sin 3 x dx  I ( x)  C , where I(x) does not contain a constant term, find
π
the value of I   .
6
Enter your answer as a whole number.

π

9. If sec x (8 2 tan x  7sec x)dx  I ( x)  C , find the value of I   .
4
Enter your answer as a whole number.

AP Grid

1. A particle is moving on the x-axis. Its velocity at any time t is given by


v(t) = 4 cos( t)  2t. What is the position of the particle at t = 4 if its position at
t = 1 is x(1) = 69?

Level N | 24
Calculus I

Section 5.2 The Definite Integral

Basic

m  2m  1 .
25
3
1. Evaluate
m 1

2. Estimate the area under the curve f ( x)  x 2  2 x from x = 0 to x = 1.


Use 4 equal subintervals and choose the right endpoint of each interval.

3. Estimate the average value of g  x   x2  1 over the interval [0, 2] using 4


subintervals and the midpoint of each interval.

2
4. Evaluate   3x  2  dx using the definition.
1

5. Find the area under the curve y  2x  4 and above the x-axis from x = 0 to x = 4
using the definition and using the area of a trapezoid.

Term 1 Week 8

1 2 1 2
2
6. Given  u ( x)dx  2 ,  u ( x)dx  1 ,  v( x)dx  , and  v( x)dx  2 , find
0 0 0
3 1
2
 2 
2 1
1
a)  3u ( x) dx b)  2u ( x ) dx c)   u ( x)  v( x) dx
2 2  3 2 
0

1
7. Show that 
0
1  x 2 dx  1 .

2
8. Evaluate   x 2  x  dx using the definition.
0

Level N | 25
Calculus I

AP Grid

1. The table below shows the recorded velocity of a particle for selected values of
time, t, over the interval 0  t  6 seconds.
t (seconds) 0 1 2 3 4 5 6
v(t) (meters per
10 12 14 16 18 16 14
second)
Using the midpoint Riemann sum with three subintervals, what is the approximate
distance, in meters, that the particle traveled during this period?

2. A test plane flies in a straight line with positive velocity v(t), in miles per minute at
time t minutes, where v is a differentiable function of t. Selected values of v(t) for 0
 t  20 are shown in the table below.
t (min) 0 5 10 15 20
velocity v(t)
11 13 14 11 10
(miles per minute)

Using the right-hand endpoint Riemann sum with four subintervals, what is the
approximate distance, in miles, that the plane travels during this period?

3. A stock company got a certain amount of investors per year to be interested in its
income guarantees. The selected results are given in the table.
Year 1 2 3 4 5
Engaged investors 28 15 15 13 20

Using the trapezoidal sum with four equal subintervals, what is the approximate
total amount of investors who got interested in the company’s profit during the
period of 5 years?

4. Let f and g be continuous functions with the following properties.


 g(x) = 17 – f (x)
3 7
  f ( x)dx   g ( x)dx
1 3
7

What is the value of  f ( x)dx ?


1

Level N | 26
Calculus I

5. Let f and g be continuous functions with the following properties.


 g(x) = 30 + 6f(x)
6

  f ( x)dx  12
1
6

What is the value of   g ( x)  4 f ( x) dx ?


1

6. The graph below represents the rate at which clients arrive to a government agency
every first Monday of the month. Clients are served during the 8 hours of work at a
constant rate of 25 clients per hour.
r
Clients per hour

72

48

24
t
3 4 5 6 7 8 0 1 2
Time, in hours
Based on this model, how many clients will be waiting in queue at time t = 8?

7. The velocity time graph of a particle moving on a straight line is shown below.
v(t)
20
15

10

5
t
0
1 2 3 4 5 6 7 8 9 10
5

10

What is the total distance travelled by the particle in the time interval 0 ≤ t ≤ 10?

Level N | 27
Calculus I

Section 5.3 The Fundamental Theorems of Calculus

Basic

dy
1. Find .
dx
4 x2
a) y   1  sin t dt b) y   sec t dt
3 x
x

2. Evaluate.
π
2
4x  2x  4
a) 
3 2 4

 dx b)  sec x tan xdx


 x2 0
1
 /2 π/4

 sin  csc xdx


2 2
c) xdx d)
/4 π/6

Core Grid

1. For f ( x)  x 2  2 , the point c  (0,2) for which f (c) = f av (average value) is the
k
solution of the equation c 2  , where k is a constant. Find k, and enter your
3
answer as a whole number.

3
2. Evaluate  3x x  9 dx .
2 3

π
10
1
0 sin 5x cos 5 x dx 
2
3. If , find k and enter your answer as a whole number.
k

x3
4. If f ( x)  x 5 cos π d , what is the value of f (1) ? Enter your answer as a

whole number.

5x
5. If f(x) =  15cos w dw , evaluate f   π  . Enter your answer as a whole number.
2

Level N | 28
Calculus I

AP Grid

1. The function f is defined and continuous on the closed interval [10, 10]. The graph
of f, shown below, consists of a quarter of a circle and two line segments.

Graph of f y
(5, 5)
(10, 4)

(0, 0) (8, 0) x
(10, 0) O

(5, 5)
x
If g is the function given by g ( x)  
10
f (t ) dt , then g(8) can be uniquely written in

1
the form  25  n  , where n is a positive integer. What is the value of n?
4
2. The function f is defined and continuous on the closed interval [10, 10]. The graph
of f, shown below, consists of three line segments.

10 y
8
6
4
2
x
10 8 6 4 2 0 2 4 6 8 10
2
4
6
8
10
x
For what value of x is 
10
f (t )dt  0 ?

Express your answer in decimal form with exactly two decimal places.

Level N | 29
Calculus I

3. The function f is defined and continuous over the open interval (10, 10). The
graph of f consists of three line segments as shown below. Let g be the function given
x
by g ( x)  
10
f (t )dt.

10 y
8
6
4
2
x
10 8 6 4 2 0 2 4 6 8 10
2
4
6
8
10
The graph of g has a local minimum at (m, n). What is the value of m?

4. A particle is moving on the x-axis. Its position at any time t for 0 ≤ t ≤ 30 is given
by x(t) = 2t3  87t2 + 600t + 211. At what time does the particle change direction
from moving to the right to moving to the left?

5. Let f be the function defined by y = f (x) = 3x2 + 2bx + 1 and having an average
(mean) value on the closed interval [0, 1] equal to 25. What is the value of b?
43
6. Let f be a continuous function such that  f ( x)dx  38 .
19
3

Find the value of k if 6  f 12 x  7  dx  k .


1

7. Let f and g be continuous functions with the following properties:


 3g(x) = 36 − 6f (x)
2
 12  f ( x)  2 dx  48
0
5
   f ( x)  1 dx  17
2
Find the average value of g(x) over the interval [0, 5].

Level N | 30
Calculus I

8. The function f is defined and continuous over the closed interval [0, 10]. The graph
of f consists of the polygonal line shown below. Let g be the function given
x

by g ( x)   f (t )dt.
0

f (x)
60

40

20

O 1 2 6 8 10 x
20
What is the value of g(0.083)?

9. The function f is defined and continuous over the closed interval [3, 6]. The graph
of f consists of the polygonal line shown below. Let g be the function given
x

by g ( x)   f (t )dt.
3

f (6, 3)

(3, 2) (1, 2) (5, 2)

(1, 2)
(3, 2)
The average value of the derivative of the function g, denoted g', over the interval
k
[3, 6] is , where k is an integer. What is the value of k?
6

Level N | 31
Calculus I

10. The function f is defined and continuous over the closed interval [6, 14]. The
x

graph of f is shown below. Let g be the function given by g ( x) 


6
 f (t )dt.
f (x)
(11, 5)
4

(6, 0) x
2 O 2

(0, 3)

What is the approximate abscissa of the point of inflection of the graph of g?

11. The function f is defined and differentiable over the closed interval [6, 10].
Below is the graph of f , the derivative of f.

y Graph of f 
(8, 6)

(10, 0)
(6, 0) O x

What is the value of f (8) if f (6) = 32?

12. A cylindrical can of radius 10 millimeters is used to measure rainfall. The can is
initially empty, and rain enters the can during a 7-day period. The height of water
π  π 
in the can is modeled by the function A(t )  2t  cos  t   5 , where A(t) is
5  10 
measured in millimeters and t is measured in days for 0 ≤ t ≤ 7. Based on this
model, what is the change in the height of water in the can between the times t = 0
and t = 5?

Level N | 32
Calculus I

13. A chocolate factory opens daily at 9:00 A.M. The factory processes milk at the rate
9
p(t )  t  t 2 cubic meters per hour for 0 ≤ t ≤ 8 where t = 0 corresponds to
64
9:00 A.M. On a given day, the factory receives milk at the rate
3 for 0  t  4
q (t )   cubic meters per hour. If on this day the factory starts
0 for 4  t  8
with 32 cubic meters of unprocessed milk in stock, how many cubic meters of
unprocessed milk would be left by the end of that day?

14. For 0  t  20 hours, water is being pumped into a reservoir at the rate of
1
R  t   t 2  t  2 cubic meters per hour and removed from the reservoir at the
3
rate of r(t) = 2t + 14 cubic meters per hour. If the amount of water in the reservoir
is increasing during the time interval (a, 20) and decreasing elsewhere, what is the
value of a?

15. A particle is moving on the x-axis between the time t = 0 and t = 9. Its initial
position at t = 0 is x(0) = 2. The velocity-time graph of the motion is shown
below.
v, in m/s (9, 30)
(3, 30) (8, 30)
30

20

10

0 1 4 7 t, in s
10

20
(6, 20)
What is the farthest point to the right of the origin (x = 0) that the particle reaches
over the time interval 0 ≤ t ≤ 9?

16. A particle is moving on the x-axis in a simple harmonic motion. Its velocity at
time t for 0 ≤ t ≤ 100 is given by v(t )   sin   . What is the total distance
5 t
3  3
travelled by the particle in the time interval 0 ≤ t ≤ 21?

Level N | 33
Calculus I

17. The velocity time graph of a particle moving on a straight line is shown below.
v(t)
40
30

20

10
t
0
1 2 3 4 5 6 7 8 9 10
10

20

What is the position of the particle at t = 10 if its position at t = 0 is x(0) = 20?

Level N | 34
Calculus I

Chapter 6 Applications of the Integral


Term 1 Week 9

Section 6.1 Areas

Basic

1. Find the area of the region bounded by the curve y = x2  1 and the x-axis from
x = 0 to x = 1.

2. Use integration to find the area of the region bounded by the curve y = −x + 1 and
the y-axis from y = 1 to y = 3.

3. Find the area of the region bounded by the curves y = (x + 1)2, y = x + 2,


the y-axis, and x = 1.

4. Find the area of the region bounded by the curves y = (x + 1)2, y = 4, the y-axis, and
x = 3.

5. Find the area of the region bounded by the curves y = x(x + 2) and y = x from
x = 2 to x = 0.

6. In the diagram below the shaded region is bounded below by the lines y = 1 and
y = 2x and above by the curve y = 2 x .

a. Find the area of the shaded region using integration with respect to x.
b. Find the area of the shaded region using integration with respect to y.

Level N | 35
Calculus I

AP Grid

1. The area of the region bounded by the x-axis, the curve (C) given by y = x2, and the
1
tangent to (C) at (1, 1) is , k is a whole number. What is the value of k?
k

2. What is the value of k when the area of the shaded region below is expressed as
 1 3
k     ?
 3  
y

2
y = 2cos(πx/2)

x
0 1 3
y = −1
−1

−2

 x 
3. What is the area of the region bounded by y = 30cos   , x  , and the
3 2
coordinate axes?

 x  1 and the line y = k  x  1 , 1  x  3, form two regions; one


1 3
4. The curve y =
4
is below the line while the other is above the line.

y 1
 
3
y x 1
4
2

1
 
y  k x 1

x
0 1 3
For what value of k are the two regions of equal areas?
Express your answer as a fraction in lowest terms.

Level N | 36
Calculus I

1
 x  2  and the line
4
5. Find the area of the region bounded by the curve y =
16
1
y =  x  1 , 0  x  2. Express your answer as a fraction in lowest terms.
2

k
6. The area of the region bounded by the curve y = 2x3 and the line y = 6x − 4 is .
2
What is the value of k?

Section 6.2 Volumes of Solids of Revolution

Basic

x 1
1. The finite region R bounded by the curve given by y  f ( x )  , the x-axis,
x3 / 2
and the lines x = 1 and x = 2 is rotated completely about the x-axis. Calculate the
volume of the generated solid.

2. A region is bounded by the curve y  f ( x )  2 x and the line y = 2x.


In each of the following rotation, calculate the volume of the generated solid:
a) The region is rotated completely about the x-axis
b) The region is rotated completely about the y-axis.

3. The region bounded by the lines y = x, y = 1, and the y-axis is rotated completely
about the line y = 2. Calculate the volume of the generated solid.

4. The region bounded by the curve y = x2 and the line y = 1 between x = 0 and x = 2
is rotated completely about the x-axis. Calculate the volume of the generated solid.

5. The region bounded by the curve y  3 x , the line x = 1, and the x-axis is rotated
completely about the y-axis. Calculate the volume of the generated solid.

6. The region bounded by the lines y = x, y = 1, and the y-axis is rotated completely
about the line x = 2. Calculate the volume of the generated solid.

7. In the diagram below, the shaded region is revolved about the x-axis.

Level N | 37
Calculus I

y  2

x=2
x(y) = 2  y2

O 2 x

Find the volume of the generated solid using


a) the shell method b) the washer method

8. A solid lies between planes perpendicular to the x-axis at x = 0 and x =  . The


cross-sections perpendicular to the x-axis between x = 0 and x =  are circular
disks with diameter in the xy-plane extending from the x-axis to the curve
x
y  cos . Find the volume of the solid.
2

Core Grid

1. The region bounded by the curve y  x2 x and the line y  2 x is rotated



completely about the x-axis. The volume of the solid generated is , where k is a
k
whole number to be determined. Give the value of k.

2. The region in the xy-plane above the x-axis and bounded by the graphs of
53
y= x and x = 8 is rotated a full rotation about the x-axis to generate a solid
4
with volume equal to k, where k is a constant. Find k, and enter your answer as a
whole number.

3. The volume of the solid generated by the rotation of the area under the graph of the
1 5π
function y = , x  [2,3] a full rotation about the x-axis is equal to , where k
x3 k
is a constant. Find k, and enter your answer as a whole number.

4. A solid is bounded by two planes perpendicular to the x-axis at x = 2 and x = 2.


The cross section of the solid with a plane perpendicular to the x-axis is a circle
whose diameter in the xy-plane is bounded by the graphs of y = x2 – 4 and

y = 4 – x2, where x   2, 2 . The volume of this solid is equal to , where k is a
15
constant. Find k, and enter your answer as a whole number.

Level N | 38
Calculus I

AP Grid

1. When the region enclosed by the graph of y  2 x  2 for x  2, the vertical line
x = 12, and the x-axis, is rotated around the x-axis, it generates a solid whose
volume is k. What is the value of k?

 
2. The region enclosed by the graphs of y  4sin  x  , y  ( x  8) , and the x-axis
8 
for 0  x  8 is rotated around the x-axis.
y
8
y = –(x – 8)
4
y = 4sin(x/8)
2
x
O 2 4 6 8

The volume of the solid it generates is k . What is the value of k?
3

3. The region enclosed by the graph of y = x2 + 4x and the x-axis is rotated around
k
the line x = 6. The volume of the solid it generates is . What is the value of k?
3

4. The region bounded below by the line y  x  1 and above by the


curve y  2( x  1) is the base of a solid. For this solid, each cross section
perpendicular to the x-axis is a square with one side lying in the coordinate plane.

y
5

2 y  2( x  1)

1
y=x– 1
x
0 1 2 3 4 5
k
Suppose the volume of the solid is expressed as . What is the value of k?
30

Level N | 39
Calculus I

14 2
5. The region bounded below by the line y = 0, above by the curve y = x and to
9
the right by the line x = 3 is the base of a solid. For this solid, each cross section
perpendicular to the x-axis is an equilateral triangle with one side lying in the
coordinate plane.
y

14
12

x
0 1 2 3
k 3
The volume of the solid is . What is the value of k?
5

Level N | 40
Calculus I

Term 1 Week 10

Chapter 7 Logarithmic, Exponential, and Inverse Trigonometric Functions

Section 7.1 The Natural Logarithm

Basic

dy
1. Find in each case.
dx
a) y = x2 + x + csc x  ln | 2x | b) y  ln  x
2. Express (2ln x  3ln y  5ln z) using a single ln function with coefficient 1.

5
A3 B 2 A
3. Express ln in terms of ln A, ln B, and ln C. A, B and C represent positive
C 2 B3
real numbers.

4. Express ln 2,592 in terms of ln 2 and ln 3.

5. Graph the function y  ln 1  x  .

 1  2x 
6. Solve ln    0.
 x 1 

7. Evaluate each of the following integrals.


5
x 1
2
dx
a) 1 3x b)  x2  2 x dx
3

8. Integrate  cot xdx.

x tan 2 x
3
9. Given y  , find y′.
1
1 2
x

Level N | 41
Calculus I

Core Grid

2
6 x2
1. The result of the integral  3 dx may be reduced to the form ln k, where k is a
0 x 2
constant. Find k and enter its value as a whole number.

Term 1 Week 11

Section 7.2 Inverse Functions

Basic

1
1. Consider the function g given by g ( x)  . Verify the rule for finding the
x 1
derivative of the inverse function at the point (0, 1).

9 d 1
2. If h given by h  x   x 4  3x3 , x  , find h ( x) .
4 dx x   8  h(2)

Section 7.3 The Exponential Function

Basic

1. Simplify.
ln e sin x
a) ln e 3 x b) e 3 ln x
c)
cos x

e x e y  3
2. Solve:  .
ln x  ln y  ln 3  ln 2

dy
3. Given y, find .
dx
a) y  e 3 x 5 
5x
b) y  e  c) y  xe cos x

4. Draw the graph of y  e x 1  1 .

Level N | 42
Calculus I

5. The function f is given by f : x  e3x , x  R .


a. State the range of f.
b. Find the inverse function, f 1 .
1
c. Sketch the curve representing the graph of y  f ( x) .

x
e
6. Evaluate  x
dx .

Core Grid

 6e cot y  π


1. If   2  dy  I ( y )  C , what is the value of I   . Enter your answer as a
 sin y  2
whole number.

ln10
3e x
2. The result of the integral 0 e x  2 dx may be reduced to the form ln k, where k is
a constant. Find k and enter its value as a whole number.

ln 8

 3e dx  a , what is the value of a? Enter your answer as a whole number.


x
3. If
ln 4

4. The result of the integral  5 x e dx can be reduced to the form ea  b , where a and
4 x 5

0
b are constants. Find the value of a + b and enter your answer as a whole number.

AP Grid

1. The amount A of certain nutrient in soil samples from a particular locale as a


 
function of time t is given by A  t   2 t 2  200e 0.05t . What numerical value do you
obtain for the rate at which the amount A is changing when t = 20 if you use
1
 0.4?
e

2. A particle moves along the x-axis with velocity at time t  0 given by


v(t )  6  78et . The particle changes direction at t = ln k. What is the value of k?

Level N | 43
Calculus I

3. A particle moves along the x-axis with velocity at time t  0 given by

v(t )  10  2et . The total distance traveled by the particle over the time interval

0  t  8 can be expressed as k  2e8  20ln5 . What is the value of k?

4. Given a differentiable function f such that f (1) = ln 3 and f ′(1) = 2. The equation of

the line tangent to the graph of y = e f  x2 at the point of abscissa x = 1 is


y = 6x + b. What is the value of b?

5. Given a differentiable function f such that f (−4) = ln12, f ′(−4) = −8, f (6) = ln101,
and f ′(6) = 4. Let g(x) = e f ( x ) . What is the average rate of change in g, the
derivative of g, over the interval [−4, 6]?


e0.64t t 3  336 for 0  t  4
6. Consider the function r given by r  t    .

 3ke 0.39 t
for t  4
For what value of k is r continuous at t = 4?
Use the approximation e = 2.7 when reporting your final answer.

7. A 4,000-liter tank is filled to capacity with water. At time t = 0, water begins to

drain out of the tank at a rate modeled by r  t   100e0.1t , measured in liters per

hour. How many liters of water remain in the tank when t = 10 hours? Use the
approximation e1 = 0.368 when reporting your final answer.

8. At time t = 0, a landfill contained 2,300 tons of solid waste. The amount of solid
 1
t 
waste stored at the landfill increases at the rate W  t   40 1  e  , measured in
12

 
tones per year. Find the average rate at which the solid waste is increasing at the
landfill between time t = 0 and time t = 12 years. Use the approximation e = 2.72
when reporting your final answer. Give your answer to the nearest whole number.

9. Garry rides his bicycle along a straight road from home to school. His velocity is
2
modeled by the function v  t  given by v  t   9e , measured in miles per minute.
t
5

Level N | 44
Calculus I

5 k
The time A, in minutes, when the distance traveled is 7 miles, is ln . Find the
2 45
value of k.

10. Let R be the region in the first quadrant bounded by the x-axis, the vertical line

x = 2, and the curve y = 2ln  6  x  as shown in the figure below.

y
4

2
R
1
x
0 1 2 3 4 5
The area of the region R is  k ln 2  6 . What is the value of k?

11. What is the area of the region R bounded above by the line y = 150, to the right by
 
the line x = 2, to the left by the x-axis, and below by the curve y = 20 e x  1 as
shown in the figure below.
y
150

x
0 2
Give your answer to the nearest multiple of 10.

12. Find lim  et  127  .


t 

13. Given the function P(t) = 2 + 5ekt with P(1) = 117. If k = ln n, what is the value of
n?

Level N | 45
Calculus I

Section 7.4 a x and log a x

Basic

dy
1. Given y, find .
dx
sin x
1
a) y  3 x 2
b) y    c) y  5  x

2

2. Integrate:
1
4 x x
5 2
a) 
1 x
dx b) x 2
dx

Term 2 Week 1
3. Given y  log 2 x 2  2 x  1 , find
dy
.
dx

 x 
4. Given y  log 4  2  , find dy .

 x 1 dx

5. Given y  1  x log 1  x  , find


dy
.
dx

1 1
6. Integrate  x log 8  dx .
 x

Core Grid

1. If f ( x)  7 log 2 (2 x 4  x 2  6) , find f (1) . Enter your answer as a whole number.


e

9
2. If  3x dx  f ( x)  C , find f (3) . Use as an approximate value for ln 3 and enter
8
your answer as a whole number.

Level N | 46
Calculus I

Section 7.5 Inverse Trigonometric Functions

Basic

1. Simplify.
a) sin cos 1  1  b) sin  cos 1 1
  1    1 
c) cos sin 1    d) cos sin 1  
  2    2 

2. Find sin a and cos a if a  tan 1  4 .

3. Find sin a and tan a if a  cos 1  0.2 .

dy
4. Find .
dx
a) y  cos 1 ln x  b) y  cot 1 2  x 
 e x  ex   e x  ex 
c) y  sec 1   d) y  csc 1  
 2   2 

Term 2 Week 2
5. Evaluate.
2 3
1 1
a) 
0 16  x 2
dx b)   3  4x  x 2
dx

10
8x dx
c)  16  x
0
4
dx d)  9x 2
 12 x  8
dx
e) x x 4 16

Core Grid

1. Consider the function f given by y  f  x   2 cos e 2 .


1

Which of the following is true about the derivative of f?


Select all that apply and enter their labels in the same order as they appear
(ascending order).
Enter the labels without any spaces or commas.
e x / 2
1. Its rule is y  .
1  ex

Level N | 47
Calculus I

ex / 2
2. Its rule is y  .
1  ex
ex / 2
3. Its rule is y   .
1  ex / 2
4. It is defined for all 2  x  2.
5. It is defined for all x < 0.

1
2. If f ( x)  tan 1 x3 , then f (2)  1 , where k is a constant. Enter the value of k as
12 k
a whole number.

1
6 x2
3. Which of the following is equal to the value of the integral0 1  x6 dx ?
Enter the labels of the correct answers in an ascending order.
1. 2tan1 1
1
2. tan 1
2
3. 2
4. π
2
5. 1
2
6. 1

1/ 4
 1 
4.  dx  , where k is whole numbers to be determined. What is the value
 1  16 x 2 k
0
of k?

 dx  ax  b 
5.   sin 1    C , where a, b, and d are whole numbers to
 143  24 x  x 2  d 
be determined and C is the constant of integration.
What is the value of ax  b when x = 100?
d
(Ignore the fact whether the original integrand is defined for x = 100 or not.)

k
6. If y  8sin 1   then y  13  y x 13 
5
, where k is a whole number to be
 x 39
determined. Give the value of k.

Level N | 48
Calculus I

7. 
dx 1  bx  1 
  tan 1    C , where a, b, and d are whole numbers to be
 81x  18x  5 a
2
 d 
determined and C is the constant of integration.
What is the value of bx  1 when x = 3?
d
Hint: Determine b and d and substitute 3 for x in the expression bx  1 , and
d
simplify. The answer is a whole number.

Level N | 49
Calculus I

Term 2 Week 3
Section 7.6 Limits of Functions

Basic

1. Find.
10 x sin 2 4 x ln x
a) lim 1 b) lim c) lim
x  0 tan x x 0 x x 1 x 1
tan x  1
x
e ln sin 2 x
d) lim  e) lim 4 f) lim
x   sec x  1
  x  x x  ln sin x
2
x
 1 1 1 
g) lim 1   h) lim x sin x i) lim   
x  x x 0 
x 0 x sin x 

Core Grid

x
1. If lim 1   = a, which of the following is true about the value of a?
2
x   x
Select all that apply and enter the labels of the correct choices in an ascending order
without spaces or separators.

1. 7 < a < 8
2. 4 < a < 5
3. 0 < a < 1
2
4. a = e
5. a = e2
6. a = 0
7. a = 1

Section 7.7 Differential Equations

Basic

1. Solve the initial-value problem y  8sin 2 x , y(0) = 4, and y′(0) = 6.

dy
2. Solve the initial-value problem  2 x 4  y 2 , y(0) = 0.
dx

3. Solve the differential equation: xy  2 y  3x 2  2 x .

4. Suppose that 500 mg of thorium-234 are reduced to 410 mg in one week.

Level N | 50
Calculus I

a. What is the half-life of thorium-234?


b. What would 500 mg of this material be reduced to in 30 days?
Core Grid

1. Consider the differential equation dy  14 x 25  y 2 , y  0 when x  0.


dx
The particular solution of this equation is y  a sin bx 2 , where a and b are constants
to be determined. Give only the value of b.

2. The integrating factor for xy  5 y  x7 is axb , where a and b are constants to be
1
determined. Give the value of the integrating factor that corresponds to x = .
3

7
3. Consider the differential equation xy  4 y  x 2 , y  when x  1.
6
k
The value of y when x  2 is . What is the value of k?
48
Hint: Find the particular solution, substitute 2, and then simplify.

dy x
4. Consider the differential equation  , y  1when x  1.
dx 3 y
The particular solution of this equation has the form  A  Bx2 , where A and B
1 k
are constants to be determined. The value of A  Bx2 when x  is . What is
4 48
the value of k?

5. Which of the following represent a slope field for the differential equation
dy
  y  1 cos πx at the six indicated points?
2
dx
Select all that apply and enter their labels in the same order as they appear
(ascending order).
Enter the labels without any spaces or commas.

Level N | 51
Calculus I

Term 2 Week 4

1. y 2. y 3. y 4. y
1 1 1 1
x x x x
1 1 1 1 1 1 1 1
1 1 1 1

5. y
1
x
1 1
1

AP Grid

1. Consider the curve whose equation is given by y = f (x) where y is a solution to the
dy x2  y 2
differential equation 4 .
dx x y
Assume further that f (−3) = 4. If (3, k) is a point on the tangent to this curve at
(−3, 4), what is the value of k?
[Hint: find the equation of the tangent to the curve at (−3, 4) and substitute 3 for x.]


2. Let f be the function satisfying f ( x)  8 x 2  x   f ( x)  1
for all real numbers x,
where f (4) = −6. Find f (4).

dy
3. Consider the differential equation  2 xy .
dx
Let y = f (x) be the particular solution to the differential equation that passes through
(0, 3). In exact form, f (5) = men, where m and n are two whole numbers to be
determined. Give the value of m × n.

dy
4. Consider the differential equation  3x 2 e y .
dx
Let y = f (x) be the particular solution to the differential equation that passes through
(0, 3). In exact form, f  4e   3  ln k , where k is a whole number to be determined.
Give the value of k.

Level N | 52
Calculus I

Chapter 8 Integration Techniques

Section 8.1 Trigonometric Expressions

Basic

1. Evaluate.
a)  sin3 x cos3 x dx b)  cos
4
x sin 4 x dx
4 3 10
dx dx
c)  x 16  x 2
d)  x x 2  25
4 5 2

2. Verify the integration rule  sec zdz  ln sec z  tan z  C .

Core Grid

1 π 1
 2 cos
2 xdx  I ( x)  C , then I    , where k is a whole number.
3
1. If
4 k
Find the value of k.
/ 6
k
2. 12 
 /12
sin 3 4 xdx 
4
, where k is a constant to be determined. What is the value of k?

1 1
3.  cos2 12 x dx  x  sin 24 x  C , where m and n are constants to be determined
m n
while C is the constant of integration.
What is the value of m  n?

Level N | 53
Calculus I

 x2
4. Consider the integral  dx .
 100  x 2
Which of the following is true?
Select all that apply and enter their labels in the same order as they appear
(ascending order).
Enter the labels without any spaces or commas.

1. The substitution x  sin10 is helpful in evaluating the integral.


2. The substitution x  10sin  is helpful in evaluating the integral.
3. Applying a proper substitution (not necessarily as in 1. or 2. above) reduces the
integral to  100 tan 2  d .
4. Applying a proper substitution (not necessarily as in 1. or 2. above) reduces the
integral to  100sin 2  d .
 x2 x x
5.  dx  50sin 1  100  x 2  C
 100  x 2 10 2


5. 128 cos 2 2 x sin 2 2 xdx  mx  n sin kx  C , where m, n, and k are constants to be
determined while C is the constant of integration.
What is the value of m  n  k?

Term 2 Week 5

Section 8.2 Partial Fractions

Basic

1. Expand using partial fractions.


x2  1 x2  5 4 x3  x  4
a) b) c)
x  x  2  x  3 x x2  9   x  x  1 x  1

4x  7
2. Given f  x   .
x  x  1
2

A B C
Express f (x) in the form   , where the constants A, B, and C are
x x  1  x  12
to be determined.

48
3. Expand .
x2  x  2
2

Level N | 54
Calculus I

4. Integrate.
2 x 2  5 x  17
2 3
4 8
a)  dx b)  3 dx c)   x  1  x 2  9  dx
1 
x x  4 x  x  1 2

Core Grid

3x3  3x  2 A B C
1. If  3   , what is the value of (A + B + C)?
x  x  2  x  1 x x  2 x 1
Enter the answer as a whole number.

2.  2x  7 b
 2 dx  a ln x  5   C , where a and b are constants to be
 x  10 x  25 x5
determined while C is the constant of integration.
What is the value of a  b?

Section 8.3 Integration by Parts

Basic

1. Evaluate.
 4
a)  x cos 2 x dx b)  ln xdx
0

c)  e x sin 2 xdx d)  e4 x sin 3 xdx

Core Grid

1. If
  3x  2 x  1 e x dx  I ( x)  C , what is the value of I(1) rounded to the nearest
2

whole number?

2. Using the rounded values cos 4 = 0.7 and sin 4 =  0.8, evaluate the integral
4
011x sin x dx , and enter your answer as a whole number.

Level N | 55
Calculus I

Section 8.4 Numerical integration

Core Grid

1. Which of the following is closest to the estimated value of the integral


I   cos  x 2  1 dx obtained using the trapezoidal formula with n = 4?
1

0
Enter the label of the proper choice.
1. 0.2
2. 0.3
3. 0.4
4. 0.5
5. 0.6
6. 1

2. Which of the following is closest to the error bound on the approximation of the
1

 
integral I   6 x  3x  10 dx obtained using the trapezoidal formula with n = 9.
3 2

0
Enter the label of the proper choice.
1. 0.02
2. 0.03
3. 0.05
4. 0.06
5. 0.07

Level N | 56
Calculus I

Term 2 Week 6

Section 8.5 Improper Integrals

Basic

1. Evaluate each improper integral. If the integral diverges, state so.


 1 
dx dx dx
a)  b)  3 c)  3
1 5 x 0 2 x 1 x x


dx
2. Determine whether the integral   x  4
5 x4
converges or diverges.

1
dx
3. Find the values of   1 for which the integral  (1  x)
0
converges and those for

which the integral diverges.

4. Determine whether the improper integral converges or diverges. If it converges,


find its limit.
1 9
dx dx
a)  10 b) 
0
x 0 6 x

1 0 1 1
dx dx dx dx
5.  4 is defined as
1
x 1 x4  0 x4 . Determine whether x
1
4
converges or diverges.

4 2 4
dx dx dx
6.   x  2
0
2
is defined as   x  2
0
2

2  x  2
2
. Determine whether the improper

integral converges or diverges.

Level N | 57
Calculus I

Chapter 9 Analytic Geometry in Space

Section 9.2 Products of Vectors

Basic

1. Given u = 3i + 5k and v = 2i + 3j  2k. Find u  v and the angle between u and v.

2. Find the cross product of the vectors u and v where u  2j  3k and
v  i  3j  5k .

3. Find the area of ABC given that A(1, 1, 0), B(0, 1, 1), and C(0, 1, 2).

4. Find the volume of the tetrahedron ABCD with vertices A(2, 0, 1), B(1, 1, 0),
C(0, 1, 3), and D(1, 1, 5) using the vectors BA , BC and BD .

5. Evaluate p   q  r  where p  i  2j  2k , q  2i  j and r  2i  j  3k .

Core Grid

1. Given p = i – 3j + 2k and q = 3i + j – 2k. If  is the angle between p and q then


a
cos   . What is the value of a?
14

2. Given u = 2i + 2j + k and v = 5j – 12k. The cross product of the vectors u and v is


equal to u  v = ai + bj + ck. What is the value of b?

3. Given A(2, 1, –4), B(4, 2, 2), and C(1, 3, –6). If s is the area of ABC, what is the
value of 2s?
4. Given A(0, –2, 3), B(3, 1, –5), and C(–1, 0, –2), and D(–1, –2, 0). The volume of
k
the tetrahedron ABCD is equal to . What is the value of k?
3

Level N | 58
Calculus I

Section 9.4 Lines in Space

Basic

1. Find the equation of the line that passes through M(0, 2, 3) and parallel to
u  2 j  3k . Give the vector form and parametric form.

2. Find the Cartesian equation of the line (k) that passes through B(2, 0, 1) and
parallel to v  2 i  2 j  8 k .
3. Find the symmetric form of the equation of the line (l) that passes through
A(2, 0, 0) and parallel to u  2 j  3k . Name the coordinate plane that (l) is parallel
to.

4. Find the symmetric form of the equation of the line (l) that passes through
B(0, 1, 5) and parallel to u  2 k .

5. Find the direction cosines of the line (l) given by


r(t )   2 i  3 k   t  3 i  2 j , t  R .

 x  5  2t  x  1  s
 
6. Given (l):  y  0 , t R and (d): y  4 , sR .
 z  3t  z  1.5s
 
Show that (l) is parallel to (d).

 x  3  2t  x  9  4s
 
7. Given (l):  y  2  t , t R and (d):  y  8s sR .
z  5  z  11  3s
 
Show that (l) is orthogonal to (d).

8. Determine whether the lines given by vector equations


r(s)   4 i  k   s  3 i  2 j  k  and r(t )   i  3 j  t  5 i  j  4 k  intersect.

9. Find the equation of the common perpendicular of


x  5  x  2  4s
 
(l):  y  6  2t , t  R and (d):  y  8 , sR .
 z  18  5t  z  9  3s
 

Level N | 59
Calculus I

 x  4  4t

10. Determine the position of the line (l):  y  8  5t , t  R relative to the plane
 z  12  6t

(P): 3x  2 z  12 .

Core Grid

1. Consider the vector equation of the line l that passes through A(1, 2, 1) and parallel
to u = 3i + j + 5k. Suppose that the point (4, 3, 6) corresponds to t = 1 and the point
(7, 4, 11) corresponds to t = 2 then what is the x-coordinate of the point on l that
corresponds to t = 10?

[Hint: Find the vector equation of l as you normally do and substitute 10 for t.
The points corresponding to t = 1 and t = 2 are given to ensure a unique answer.]

2. The parametric equations of the line passing through the point P(7, 0, –4) and
parallel to the vector u = i + 4k are x = p + qt, y = 0, and z = r + st, where p, q, r,
and s are constants. If the point Q (a, 0, 0) lies on this line, find the value of a using
the parametric equations after determining the corresponding constants.

3. Consider the line (l) that passes through A(0, 1, 2) and B(2, 2, 5). If C is a point on l
whose x-coordinate is 20, then what whole number must be equal to its y-
coordinate?
Find the Cartesian equation of l and substitute 20 for x and solve for y. Simplify
your answer.

x  1 t

4. The distance from A(1, 1, 0) to  l  :  y  3  t , t  R, is a.
 z  1  2t

What is the value of a?

x2 y3 z 4
5. The symmetric form of the equation of a line is   .
2 3 7
If (12, b, c) is a point on this line, what would the value of c be?

6. Given the line (l) that is parallel to the vector v = 3i + 2j and passes through the
point A(3, –2, 6).
If (9, b, c) is a point on this line, what would the value of c be?

Level N | 60
Calculus I

7. A line (l) is parallel to y-axis and passes through the point A(12, 7, 10).
If (a, 11, c) is a point on this line, what would the value of a + c be?

8. Which of following is true about the line r(t) = (–i – 4j + 3k) + t(–i + 3j + 5k),
t  R?
Select all that apply and list the numbers in the same order as they appear
(ascending order) without spaces or commas.
1. The line makes an acute angle with the positive x-axis.
2. The line makes an obtuse angle with the positive x-axis.
3. The line makes an acute angle with the positive y-axis.
4. The line makes an obtuse angle with the positive y-axis.
5. The line makes an acute angle with the positive z-axis.
6. The line makes an obtuse angle with the positive z-axis.

9. The line (l) is given by r(t) = (–2i + j – 2k) + t( 3 j + k), t  R. The angle between
(l) and the positive y-axis equals k. What is the value of k?
x 1 y  3 z  5
10. Which of the following lines are parallel to the line (l):   ?
2 3 4
Select all that apply and list the numbers in the same order as they appear
(ascending order) without spaces or commas.
x3 y  2 z 6
1.  
1 2 7
x y2 z4
2.  
6 9 12
3. r(t) = (i + 2j – 4k) + t(8i – 12j + 16k), t  R
4. r(t) = (i – 2j + 7k) + t(3i – 2j), t  R
 x  2  2t

5.  y  4  3t , t  R
 z  1  4t
 x  1  3t

6.  y  2 , t  R

 z  7  6t

Level N | 61
Calculus I

 x  2  4t
11. Which of following lines are perpendicular to the line (l):  y  3 , t  R?
 z  7  t
Select all that apply and list the numbers in the same order as they appear
(ascending order) without spaces or commas.
x3 y  2 z 6
1.  
1 2 7
x z4
2.  ,y2
1 4
3. r(t) = (i + 2j – 4k) + t(–9i – 8j + 6k), t  R
4. r(t) = (i – 2j + 7k) + t(–i +2j – 4k), t  R
x  2  t

5.  y  4  4t , t  R

 z  1  4t
 x  1  16t

6.  y  2 , t  R

 z  7  4t

12. Which of following statements is true about the relative position of the lines
 x  1  2t  x  1  s

 l  :  y  2t , t  R and  m  :  y  3  2s , s  R ?

z  1 t  z  1  2s
Select all that apply and list the numbers in the same order as they appear
(ascending order) without spaces or commas.
1. (l) and (m) are intersecting.
2. (l) and (m) are skew.
3. (l) and (m) are perpendicular.
4. (l) and (m) are parallel.
5. (l) and (m) are coincident.
6. (l) and (m) are not intersecting.

x  1 t  x  3  2s
 
13. Given the two lines  l  :  y  2t , t  R and  m  :  y  2  2s , s  R intersect
 z  2  t 
 z  1  s
at (a, b, c). What is the value of c?

x  1 x  2  s
14. Given two skew lines (l):  y  5  t , t  R and (m): 
 y  0 , s  R. The
 z  t 
z  1
distance between (l) and (m) is equal to k . What is the value of k?

Level N | 62
Calculus I

15. Consider the plane Q whose equation is given by 2x + y + 2z = 2 and the line l
 x  3  5t

whose equation is given by  y  2  2t , t  R .
z  5

- If l is parallel to Q, enter 111 for your answer.
- If l is contained in Q, enter 999 for your answer.
- If l intersects Q, give the y-coordinate of the point of intersection.

16. Consider the plane Q whose equation is given by 6x + 2y + 3z = 5 and the line l
 x  10  2t

whose equation is given by  y  4  6t , t  R .
z  3

- If l is parallel to Q, give the distance between them.
- If l is contained in Q or l intersects Q, enter 0 for your answer.

Level N | 63
Calculus I

Chapter 10 Complex Numbers

Term 2 Week 7

Section 10.1 Algebraic Form of a Complex Number

Basic

1. Simplify 12(5 + 3i)  6(7  9i).

2. Simplify (3 + 5i)(8  3i).

3. Solve x2 + 8x + 25 = 0.

4. Given z = 5  12i.
a. Find the conjugate of z.
b. Find the modulus of z.
c. Find the product of z with its conjugate.

5. If z = 6 + 5i and w = 4 + 3i, find z ÷ w.

Core Grid

1. Consider two complex numbers z  4  ai and w  ai  3x  10 , where a is a real


number. If z = w, what is the value of x?

2. Which of the following is equal to i8n 4 for any natural value of n?


Select all that apply and enter their labels in the same order as they appear
(ascending order).
Enter the labels without spaces or commas.
1. i7
2. i8
3. (–1)2n
4. –1
5. 1
6. i2n

3. Given two complex numbers z  2  3i and w  6i  5 , what is the value of


Re(z  w)?

4. Given two complex numbers z  5i  3 and w  6  i , what is the value of


Im(z + w)?

5. What real number is equal to 3z  4w if z  4i and w  2  3i ?

Level N | 64
Calculus I

6. If z  4  5i and w  6  3i , what is the value of Im(zw)?


7. Which of the following is a root of the equation x2  2x  10  0 ?
Select all that apply and enter their labels in the same order as they appear
(ascending order).
Enter the labels without spaces or commas.
1. –5
2. 1  3i
3. 11  2
4. 1  3i
5. 1  3i
6. 3i  1

8. Which of the following pairs are conjugates?


Select all that apply and enter their labels in the same order as they appear
(ascending order).
Enter the labels without spaces or commas.
1. 8i and 8
2. 7 and –7 + i
3. –5 + 6i and –5 – 6i
4. 3i + 2 and 3i – 2
5. –4i and 4i

9. What is the modulus of 8  6i ?

10. If    
3i  9  2  3i  a  bi , what is the value of a?

Level N | 65
Calculus I

Term 2 Week 8 and week 9

Section 10.2 Trigonometric (Polar) Form of a Complex Number

Basic

1. Express z  20 sin 60  i cos60 in polar form.

2. Given z  10  10 3i and w  1  i .
a. Express z and w in polar form.
b. Find zw. Express your answer in polar form.
z
c. Find . Express your answer in polar form.
w

3. Given z  1  i .
a. Express z in polar form.
b. Find z6.

4. Find the four fourth roots of z  81i . Express your answer in polar form.

5. Given z  16 .
a. Find the four fourth roots of z. Express your answer in polar form.
b. Verify that the roots form the vertices of a square inscribed in a circle.

6. Compute the values of ( i )3 4 . Express your answer in polar form.

Core Grid

1. Which of the following is true about the complex number z  3  3i ?


Select all that apply and enter their labels in the same order as they appear
(ascending order).
Enter the labels without spaces or commas.

i
1. Its polar form is z  12e 3
.

i
2. Its polar form is z  2 3e 6
.
  π  π 
3. Its trigonometric form is z  2 3  sin     i cos     .
  6  6 
  π  π 
4. Its trigonometric form is z  12  cos     i sin     .
  3  3 
  π  π 
5. Its trigonometric form is z  2 3  cos     i sin     .
  6  6 

Level N | 66
Calculus I

2. Which of the following is true about the complex number z  2  sin 60  i sin 60 ?
Select all that apply and enter their labels in the same order as they appear
(ascending order).
Enter the labels without spaces or commas.

i
1. The polar form of z is z  6e 4 .
2. The polar form of z is z  2ei 60 .
 π
3. The trigonometric form of z is z  6  cos  i sin  .
π
 4 4
4. The trigonometric form of z is z  2  cos60  i sin 60 .
 π π
5. The polar form of z is z  6  sin  i cos  .
 4 4

3. Consider z  3  cos 265  i sin 265 and w  7  cos(40)  i sin(40)  .


If zw = a cis, where a is a whole number and 0 ≤  ≤ 360, then what is the sum of
a and ?

4. Consider z  20  cos120  i sin120 and w  4  cos75  i sin 75 .


If z/w = a cis, where a is a whole number and 0 ≤  ≤ 360, then what is the sum
of a + ?

5. Given z  3  cos18  i sin18 . If z5  ki , what is the value of k?

Section 10.3 Additional Topics on Complex Numbers

Basic

π
1. Find the locus of P(z) if z = x + iy and Arg( z  1  i )  .
4

z i
2. Find the locus of P(x, y) if z = x + iy and is real.
z 1

3. Form a quadratic equation whose roots are 3 + 7i and 4  i.

4. Solve  4  i  z  6i  5  8i .

1
5. Solve z 2  2iz  1  i  0 .
2

Level N | 67
Calculus I

6. Solve z 4  81  0 .

Core Grid

1. Which of the following Argand diagrams could be the locus of P(x, y) if z = x + iy


and Arg(z – i) = 60°?
Select all that apply and enter their labels in the same order as they appear
(ascending order).
Enter the labels without spaces or commas.
1. 2. 3.

4. 5.

2. The roots of the equation z2 + bz + c = 0, where b and c are real numbers and z is a
complex number, are 2 – i and 2 + i. What is the value of c?

3. What is the modulus of the root of the equation (1 + i)z – 11i = –7 + 6i?

4. Given that a + i and b + i are two roots of the equation z2 – (5 + 2i)z + 5 + 5i = 0,


what is the value of a + b?

5. The list below includes all the roots of z3 – 8 = 0. Select all the roots.
Enter their labels in the same order as they appear (ascending order). Enter the
labels without spaces or commas.
1. 2
2. 1  3i
3. 1  3i
4. 2
5. 1  3i
6. 1  3i

Level N | 68
Calculus I

6. Which of the following is true about the locus of the point P which represents the
complex number z in an Argand diagram if z satisfies | z  3  2i | = | z + i |?
Select all that apply and give their labels in the same order as they appear
(ascending order) without any commas or spaces.
1. It is a circle.
2. It is a straight line.
3. It is the perpendicular bisector of AB where A(3, 2) and B(0, 1).
4. It is the perpendicular bisector of AB where A(3, 2) and B(0, 1).
5. It is a circle with center at (3, 2).
6. It is an empty set.
7. Determine the locus of the point P which represents the complex number z in an
 z  1  7i  
Argand diagram if z satisfies Arg   ?
 z  4
- If it is a circle, give its radius.
- If it is a point, give its abscissa.
8. The locus of the point P which represents the complex number z in an Argand
k
diagram that satisfies z  6  3 z  2 is a circle whose radius is , where k is a
2
whole number to be determined. What is the value of k?
9. The equation x3  4x2 + 9x  36 = 0 has one real root. What is the value of this root
given that 3i is a root of this equation?
10. The equation x4  6x3  7x2  54x  144 = 0 has one positive root. What is the
value of this root given that 3i is a root of this equation?

Level N | 69
Calculus I

Top Questions for Chapter 3

1. Let f  x   1  cos x

a) What is the domain of f  x  ?

b) What is the domain of f   x  ?


c) Write the equation of the line normal to the curve at x 
2
  3 
d) Is Rolle’s theorem applicable for f  x  in the interval  , 
2 2 

π
2. A curve has parametric equations x  3tan θ, y  cos2 θ , 0  θ 
2
dy
a) Find
dx
π
b) Find an equation of the normal to the curve at the point where t 
4
c) Find a Cartesian equation of the curve and state its domain.

  
3. A curve has parametric equations x  cos t , y  cos  t   , 0  t 
 4 2

a) Find an equation of the tangent to the curve at the point where t 
4
b) Write the Cartesian equation of the curve.

4. A curve C is given parametrically by the equations x  3cos t ,


y  2  4sin t , 0  t  2
a) Find the coordinates of all the points at which C intersects the coordinate
axes giving your answers in surd form where appropriate.
b) Sketch the graph of C

c) P is a point where t  , find the equation of the normal to C at P.
4

Level N | 70
Calculus I


5. A curve C has parametric equations x  cos 2 t and y  2cot t , 0  t 
2
dy 
a) Find and the equation of the tangent at the point where t 
dx 6
b) Find the coordinates of the point P where the tangent found in part (a)
cuts the x-axis.

6. A curve C has parametric equations x  2cos t , y  2  2sin 2 t , 0  t   .


dy 
a) Find at the point where t 
dx 4
b) Find a Cartesian equation for C in the form y  f  x  ,  k  x  k ,

stating the value of the constant k.


c) Write down the range of f  x 

d) Sketch the graph of f  x  .

7. The position of a particle moving in the xy-plane is given by


x  t   sin  2t  , y  t   3t where    t  

a) Find the range of x  t  and that of y  t  .

b) Find the smallest positive t such that x-coordinate of the particle is a local
maximum.
c) Find the speed of the particle at the time found in b.
d) Find the distance traveled by the particle from t    to t   . Compare
the answer with 5 .
e) Sketch the graph of the path of the particle indicating clearly the direction

of motion along it.

Level N | 71
Calculus I

1
8. Consider the two function: h  t   et  1 ; k  t    ln t   1 .
2
1
a) Find the area of the region formed by the graphs of f and g between t 
3
and t  0.5 .
b) Find the volume of the solid generated when the region formed by the
1
graphs of f and g between t  and t  0.5 is revolved about the line
3
y  3.

c) For the function n  t   h  t   k  t  , find the absolute minimum and the

1 1 
absolute maximum on  ,  .
3 2

Level N | 72
Calculus I

Top Questions for Chapter 4

9. Height (meters)

f(x)

x
0
The height f of the ramp in a toy is a function of x. Given that:
(i) f (0)  0 f (0)  0
(ii) f (4) 1 f (4) 1
(iii) If 0 < x < 4 then f is increasing
a) Find, if possible, the value of the constant k  0 ; so that f ( x)  k x 2
satisfies the condition in (ii).
x2
b) Find if possible the value of the constant m 0 so that f ( x)  m x 3 
16
satisfies the condition in (ii).
c) Use the answer of part b to show that f ( x) does not satisfy the condition
(iii)
xm
d) Given f ( x)  m  0, n  0 . Find n so that f ( x) satisfies condition (ii)
n
then show that f ( x) satisfies (i) and (iii).

Level N | 73
Calculus I

10.
v(t)

15
(meters per second)
(5,10) (12,10)
Velocity

10

5
(20,0)
t
O 5 10 12 15 20
Time (seconds)
A motorcycle is traveling on a straight road for 0  t  20 seconds, the
motorcycle’s velocity v(t) in m/s is represented by the graph above.
20
a) Find  v  t  dt , give its unit and explain its meaning.
0

b) Find v  5 and v 16 if any of these values does not exist state so give the
unit of measure of v 16 .
c) Express a(t), the acceleration of the motorcycle as a piecewise – defined
function.
d) Find the average rate of change of v when 8  t  16 , knowing that
v 16  5 .
e) Does the Mean Value Theorem guarantee a value of K, K  8,16 such
that v  K  equals the average rate of change? Explain.

Level N | 74
Calculus I

11.

The graph of g   x  is given above. g   x  has a tangent of slope = 0 at x = –7,

x = 2 and x = 5 and a tangent of an undefined slope at x = 4.


In each of the following, you should justify your answer by showing all
working.
Over the interval  9,9  ,

a) Find the values of x for which g(x) has a relative minimum.


b) Find all the values of x at which g attains a relative maximum.
c) Find all values of x at which g   x   0 .

d) Find the value of x at which g attains its absolute maximum?


12. The function g   x   e  x / 2 sin  x3  is the first derivative of the function g(x).
g(0) = 0.
a) Using your graphing calculator, sketch the graph of g’(x) , 0 ≤ x ≤2
b) Using the graph of g  , what type of concavity if any, does g  x  have

when 1.3  x  1.6 . Explain.


c) Find x, 0  x  2 at which g  x  , has an absolute maximum. Explain.

d) Write the equation of the line tangent to the graph of g  x  at x = 1.

Level N | 75
Calculus I

13. The graph of f  is shown in the diagram below. For

6  x  1 f   x    x2  5x and

for 1  x  6 f  is piecewise linear.

a) For 6  x  6 , find all the values of x at which f has a relative


maximum, give reasons.
b) For 6  x  6 , find all the values of x at which f has an inflection point.
c) Find the intervals at which the graph of f is concave down.
d) If given that f  6   10 , find the absolute maximum value of

f  x ,  6  x  6 .

Level N | 76
Calculus I

14. a) Show that f  x   2 x  x3 has a root in the interval 1.3,1.4

2 xn
b) Show that f  x   0 can be written as xn1  and start with
xn

x0  1.37 to calculate x1 and x2 , giving your answer to 5 decimal


places.
c) Using the change of sign, find the value of the root of f  x   0 to 4

decimal places.

15. a) Let f  x   x3  x  1 . Use a sign change method to show that the

equation x3  x 1  0
has a root between x  1 and x  2 .
b) By taking x  1 as first approximation to this root use the Newton

_Raphson formula to find this root to 3 decimal places.

16. a) Show that the equation x3  6x  2  0 has a root


between x  0 and x  1

2  xn3
b) Use the iterative formula xn 1  with x0  0.5 to find this root
6
correct to 4 decimal places showing the results of each iteration.

17. The equation has a root between x  0 and x  1 .

a) The equation can be rearranged into the form x  g  x  where

g  x   px3  q .

State the values of p and q.

b) By considering g   x  , show that the iterative form xn1  g  xn  with a

suitable starting value converges to the root between x  0 and x  1.


[You are not required to find this root].

Level N | 77
Calculus I

Top Questions for Chapter 5

18.
Distance x (mm) 0 50 100 150 200 250 300
Diameter L(x) (mm) 20 25 20 35 28 27 35

Suppose that a water pipe is 300 millimeters (mm) long with circular cross
sections of varying diameter. The table given above gives the measurements of
the diameter of the water pipe at selected points along the length of the water
pipe, where x represents the distance from one end of the water pipe and L(x) is
a twice-differentiable function that represents the diameter at that point.
a) Give an integral expression in terms of L(x) that represents the average
radius, in mm, of the water pipe between x = 0 and x = 300.
b) Give an approximate value of your answer from part (a) using the data
from the given table and a midpoint Riemann sum with three subintervals
of equal length. Show all your working.

 L  x 
250 2

c) Explain the meaning of the expression     dx in terms of the


90 
2 
water pipe, using proper units.
d) Why there must be at least one value x, for 0 < x < 300, such that
L  x   0 .

Level N | 78
Calculus I

19.
t v(t)
(seconds) (meter per seconds)
0 0
4 10
8 18
12 25
16 50
20 60
24 73
28 82
32 70
36 50
40 73

The velocity v  t  of a train in m/s for 0  t  40 is given in the table which


gives values of v  t  at 4 second intervals of time t. x 8  12 .
a) Find an approximation for the acceleration of the train in m/sec2, at t = 28.
40
b) Approximate  v  t  dt with a Riemann sum, using the midpoints of five
0
subintervals of equal length and give a meaning for your answer.
c) Suppose that the acceleration of the particle is positive for 8  t  16 .
Explain why the position of the particle at t  16 is greater than x  156 m.
20.
t v(t)
(min) (km/min)
0 8.0
5 10.3
10 18.2
15 8.0
20 14.5
25 12.6
30 13.6
35 14.3
40 19.7

Level N | 79
Calculus I

A particle moves in a straight line with positive velocity v  t  , in km per min at

time t minutes, where v is a differentiable function of t. Values of v(t) 0  t  40


are shown in the given table.

a) Use a left point Riemann sum with four subintervals of equal length and
40
values from the table to approximate  v  t  dt . Show your working. Using
0
40
correct units, explain the meaning of  v  t  dt
0
in terms of the particle

motion.
b) Using the values in the table, is there a value of t for which the
acceleration of the particle is equal to zero on the (0, 40)? Justify your
answer.

c)  
v  t   sin t 2  t  t is used to model the velocity of a particle in km/m,

0  t  40 .
i) Calculate the distance covered in the given interval.
ii) Calculate the average velocity in the given interval.

Level N | 80
Calculus I

21. Water flows out of a cylindrical pipe at a rate F  t  given in the adjacent table.

a) Use a right endpoint of Riemann sum with 5


40 t F t 
subintervals of equal width to find  F  t  dt .
0
(hours)
(litres/hours
)
b) Give the meaning of the answer of part a in terms 0 7.8
of water flow, specify units. 4 12.5
c) Will there be a time t, t   0, 40 such that 8 12.7

F   t   0 ? Why? 12 15.2
16 18.2
d) Suppose that the rate of flow F  t  can be
20 18.5
estimated by F  t  
1
82
536  24t  t 2  . 24 12.2
28 12.8
Use F  t  to estimate the average rate of water
32 7.3
flow during the period of 40 hours. 36 10.9
40 7.8

22. The height, in feet, of a cylinder with constant radius is a twice differentiable
function h of time t where t is measured in minutes. For 0  t  14 , the graph of
h is concave down. The table gives selected values of the rate of change, h  t  ,

of the height of the cylinder over time interval 0  t  14 .

t (minutes) 0 3 5 9 11 14
h  t  6.1 3.6 3.2 2.5 2.0 1.7

Level N | 81
Calculus I

a) Use a right Riemann sum with five subintervals indicated by the data in
14
the table to approximate  h  t  dt . Using correct units, explain the
0
14
meaning of  h  t  dt in terms of the height of the cylinder.
0
14
b) Is your approximation in part (a) greater or less than  h  t  dt ? Explain.
0

23. A big rectangular reservoir of length 120 m, width 100 m and depth 50 m,
contains 1,500 cubic meters of water at t  0 .
Water is leaking from the reservoir at a rate r  t  cubic meters per hour,

t  h 0 3 6 9 12

r t  0 120 150 180 200

During the same time interval, water is poured in at a rate of

R  t   37e0.3t m3/hr.

a) Use the midpoint Riemann sum to approximate the amount of water that is
leaked during the
12 hrs.
b) Calculate the amount of water pumped in during the 12 hours.
c) What is the rate of increase of the water in the reservoir at t  6 hrs and
how fast is the level of water increasing?

4
 3 1

24. Find the exact value of  10 x 2  3x 2  dx .
1 

Level N | 82
Calculus I

25.
Expansion Temperature
in cm M  x
(x)
10 100
7 80
5 75
1 63
0 60

The table gives some values of the x, in cm, and the corresponding values of the
temperature M  x  in degrees Celsius (°C). The function M  x  is increasing

and twice differentiable.


a) Estimate M   6  . Show your working and give units of measure.

b) Give an integral expression in terms of M  x  for the average temperature.

c) Estimate the average temperature using a trapezoidal sum with the four
subintervals using the data given in the table.
10
i. Find  M   x  dx , and give units of measure.
0
10
ii. Explain the meaning of  M   x  dx
0

Level N | 83
Calculus I

Top Questions for Chapter 6

26. The function f is continuous and differentiable for 3  x  4 , and f  0  3 .

The graph of f  is given below.

2  (4, 2)

1
3 2 1 1 2 3 4
x

1

 2 
(3, 2)

a) Find f  3 and f  3

b) Find the absolute maximum of f.


c) Find the critical points for the function f and classify each.
d) Find the average value of f   x  on 3  x  4 .

27. The figure shows the graph of the curve with equation y  x ln x , x  0
y

x
1 2

Level N | 84
Calculus I

The shaded region R bounded by the line x  2 , the x-axis and the curve
a) Find the exact value for the area of R.
b) Complete the table and use trapezoidal rule to find the approximate
value for this area to 5 decimal places.
x 1 1.2 1.4 1.6 1.8 2
y  x ln x 0 0.2188 0.4711 1.3863

28. The figure shows the finite shaded region R, which is bounded by the curve

y  xe x , the line x  1 , the line x  4 and the x-axis.


y
The region R is rotated through 360 degrees about the
x-axis. Use integration by parts to find an exact value
for the volume of the solid generated.
x
1 4

29. A container has the shape of a cylinder with radius r = 5 cm. the height of water
in the container is h at a given time t (seconds).
The volume of water is decreasing at a rate 3  h cm3/ s.
dh
a) Find k if k .
dt
dh
b) Given that the height of water initially is 10 cm. solve k .
dt
c) When the container becomes empty?

Level N | 85
Calculus I

30. Given a cone with vertex pointed downwards, whose radius is 10 cm and
height double its radius. Water is pumped into the cone so that the depth of
the water h is changing at the constant rate of
1
cm/hr, and the radius of the water surface is r.
4
a) Find the volume V of water in the container when h = 8 cm.
b) Find the rate of change of the volume of water in the container, with
respect to time, when h = 8 cm.
c) Find the relation between the rate of change of the volume of water and
the surface area of the water. (Hint: directly proportional).

The figure shows part of the curve (C) with parametric equations x   t  1 ,
2
31.

1
y  t 3  3 , t  1 . P is a point on the curve where t  1 . The line l is the
2
normal to C at P.
(C)
a) Find an equation of l. l
y
The shaded region R is bounded by C, l, the x-axis, and the
P
line with equation x  1
b) Using integration and showing all your working,
x
find the area of R. 1
c) Using integration find the volume generated when
the shaded area is rotated 2 radians about the x-axis.

Given y  e3 x  x .........  C  .
2
32.

a) Find the area limited by the graph of (C) and y = 2.


b) Find the area limited by the graph of (C) and y = 1 and y = 2.
c) Find an expression for the volume generated when the region limited by
the graph of (C) and y = 2 is rotated 2 about y = 1.

Level N | 86
Calculus I

2x
33. Let R be the region in the first quadrant bounded by f  x   , 1 x  6
3x 2  2
a) Find the exact value for the area R, show all your work.
b) The line x  k divides the area R into two equal parts, find the value of k.
c) Calculate the average value of f   x  in the given interval.

34. Given the graphs of the two curves y  x 2 and y  1  x3 . The region A in the
first quadrant is bounded by y  x 2 and y  1  x3 and the y-axis and the region
B in the first quadrant is bounded by the two curves and the x-axis.
a) Find the area of A.
b) Find the area of B.
c) Find the volume of the solid generated when A is revolved about the x-
axis.
1
35. Consider the graph of the function f  x   is graphed when x  0 . If A is
x 1
the region bounded by the graph of f, the x-axis and y-axis and the line x = m;
and B is the region in the first quadrant bounded by the line x = m, the graph of
f  x  and x  3, 0  m  3 .
a) Find the area of A in terms of m.
b) Find the volume generated when A is revolved about the x-axis.
c) Find m such that the volume when B is revolved about the x-axis is half
that of A when revolved about x-axis.

36. Given h  x   x3  4 x , and m is a tangent line to the graph of h with equation

y   x  2 . Let S be the region bounded by the graph of h, the line m:


a) Sketch the graph of the given function h and the tangent line m.
b) Prove that line m, is tangent to the graph of y  h  x  at the point x = 1 .

c) Find the area of region S.


d) Find the volume of the solid generated when the region in the second
quadrant bounded by the graph of h  x  and the x-axis is rotated about the

x-axis.

Level N | 87
Calculus I

37. Given the function by g  x   x3  2 x2  3x  2  2cos x , Let R be the shaded

region in the quadrant II bounded by the graph of g, the x-axis and y-axis. S is
the region bounded by the graph of g and line t, which is tangent to the graph of
g at x  1.5 .
a) Find the area of region of R.
b) Find the volume of the solid generated when R is rotated about y = –1.
c) Without actually evaluating, write an integral expression for the area of
region S.

(A graphic calculator is needed)


38. a) Find the area of the region R enclosed by the graph of y  3 x  1 , the
vertical line x  3 , and
the x-axis.
b) When R is revolved about the horizontal line y  3 , find the volume of the
solid generated.
c) When R is revolved about the vertical line x  7 , find the volume of the
solid generated.
x3
39. Given that the region M is bounded by graphs of the curve y  , the line
2  x2
y = – x + 4 and the y-axis.
a) Sketch the given functions.
b) Find the area of M.
c) Find the volume of the solid generated when M is revolved about the line
y  0.
d) Find the volume of the solid whose base is the region M if the cross
sections taken perpendicular to the x-axis are equilateral triangles.

Level N | 88
Calculus I

40. [A graphic calculator is needed to solve this question.]


Let S be the region in the first quadrant enclosed by the graph of

y  e2 x , y  2  2cos x and the y-axis.


2

a) Find the area of the region S


b) When region S is revolved about the x-axis a solid is generated, find the
volume of this solid.
c) The region S is the base of a solid. Consider each cross section, of this
solid, perpendicular to the horizontal axis as a rectangle whose height is
three times the width, where the width is bounded by the graphs of the
given functions. Find the volume of the solid.

41. Let R be the region in the first quadrant enclosed by the graphs of y  x and

y  x3 .
a) Find the area of R.
b) The region R is the base of a solid. For this solid, at each x the cross
 x 
section perpendicular to the x-axis has area A  x   cos   . Find the
 2 
volume of the solid.
c) Another solid has the same base R. For this solid, the cross sections
perpendicular to the y-axis are squares. Find the volume of this solid.

Level N | 89
Calculus I

42. For 0  t  3.5 , water flows into tank P at a rate of P  t   2t  4  4e3sin x gallons

per hour. In the same time interval, water flows into tank Q at a rate of Q  t  .

The graphs of P  t  and Q  t  intersect at t  m and t  2.800 . Both tanks are

initially empty.
a. Find the amount of water in tank P at t  3.5 .
b. During the time interval 0  t  m , water flows into tank Q at a constant
rate of 17.6 gallons per hour. Find the difference in the amount of water in
tanks P and Q at t = m.
c. The area of the region bounded by the graphs of P  t  and Q  t  for

m  t  2.800 is 74.833.
How much water is in tank Q at t  2.800 ?

43. a) Find the area of the region enclosed by y  3cos 2 x , 0  x  and the
4
axes.
b) The region in part (a) is rotated through 2 about the x-axis, find the
volume of the solid generated.

44. a) Find the area of the region enclosed by y  3cos 2 x , 0  x  and the
4
axes.
b) The region in part (a) is rotated through 2 about the x-axis, find the
volume of the solid generated.

Level N | 90
Calculus I

Top Questions for Chapter 7

45.
t
0 10 20 30 35 45 50
(sec)
v t 
–10 –20 –10 –12 –15 0 15
(m/sec)
a t 
1 6 4 1 4 5 3
(m/sec2)

The continuous functions v  t  and a  t  of t, 0  t  50 , represent the velocity

and acceleration of a particle.


35
a) Find an estimate for
10
 v  t  dt using a trapezoidal method with three

subintervals from the given table and explain the meaning of your answer.
50
b) Calculate the exact value of  a t  dt
0
and explain the meaning of your

answer.
c) For 0  t  50 , will there be a time t when v  t   8 ? What theorem

guarantees this?
d) For 0  t  50 , will there be a time t when a  t   0 ? What theorem

justifies this?

Level N | 91
Calculus I

46. f  x  is differentiable over

The table gives the values of f and its derivative f  . bg


f  x  0 for

x   2.5,1 .

x -2.5 -2.0 -1.5 –1 0 0.5 1

bg
f x 2.75 2 1.75 2 4 5.75 8

f b x g -2 -1 0 1 3 4 5
1
a) Showing your work, find the value of  3 f   x   5 dx .
1.5

b) i) Find the equation of the line tangent to the graph of f at x = 0.5.


ii) Approximate the value of f  0.4  , using the equation of the tangent

line.
iii) Compare the approximate value of f  0.4  , to the actual value of

f  0.4  .

c) Find a positive real number r having the property that there must exist a
bg
value c with 1  c  0 and f  c  r . Give a reason for your answer.

47. v  t   ln (t 2  5t  7) is the velocity of a particle moving along the x-axis


 0  t  5 : when t 0 ;
x = 5.
a) i) Find t; 0  t  5 at which the particle changes direction.
ii) Find the time intervals when the particle moves to the left.
b) Find the acceleration of the particle at t = 2.
c) Find the average speed of the particle over the interval 0  t  2 .
d) Find the position of the particle when t = 2.

Level N | 92
Calculus I

48. A particle M moving along a straight path has velocity v  t   2 ln  3  t 2  cm/s.

a) Find the displacement of the particle for 0  t  18 .


b) Find the time 0  t  18 for which the particle’s instantaneous velocity is
equal to the average velocity over the interval.
c) Find the acceleration of the particle at t  4 .
d) A particle Q arrives at particle M’s starting position at t  9 and follows
the path of particle M. During the interval 9  t  18 Q travels in the same
direction as M with a constant acceleration of 3 cm/s2. Q catches up with
M at t  18 . Find the velocity of Q at t  9 .

2
49. a) Find the exact value of  csc xdx
4
b) i) Given y  csc x , complete the table
x  5 3 7 
4 16 8 16 2
y 1.41421
2
ii) Use the trapezoidal rule, to obtain an estimate for  csc xdx ,
4
giving your answer to 4 decimal places.
c) Calculate the % error in using the estimate you obtained in part (b).

50. At any time t  0 , in days, the rate of growth of bacteria population is given by
y  ky , where k is a constant and y is the number of bacteria present. The
initial population is 1,500 and the population doubled during the first 6 days.
a) Write an expression for y.
b) At what time t, will the population have increased by a factor of 8?
c) By what factor will the population have increased in the first 12 days?

Level N | 93
Calculus I

dy
51. Given  3 x  2 y  5.
dx
a) Find the values of a, b and c if y  a x  b  ec x is a solution of
dy
 3 x  2 y  5.
dx
d2y
b) Find .
dx 2
c) y  f ( x) is a particular solution to the differential equation with the initial

condition f  0   2 . Approximate f 1 using Euler’s method with

1
x0  0 and step size  .
2
d) Let g  x  be another solution of the differential equation with g  0  m ,

where m is a constant.
Find m if Euler’s method gives the approximation g 1  0 starting at

x0  0 with step size = 1.

Level N | 94
Calculus I

dy y  2
52. Consider the differential equation  where x  0 .
dx x2
a) Sketch the slope field for the given differential equation at the nine points
indicated.

 2  

 1  

   y
-1 1 2

b) Find the particular solution y  f  x  to the differential equation with the

initial condition f  3  1 .

c) For the particular solution y  f  x  described in part (b), find lim f  x  .


x 

dy 2   x 
53. Consider the differential equation   y  1 1  sin   .
dx   2 
a) Sketch the slope field for the given differential equation at the nine points
indicated.
y

 1 

   x
–1 O 1

 –1  

b) y = k, where k is a constant, satisfies the given differential equation, find


the value of k.
c) Determine the particular solution y  h  x  to the differential equation
given that h 1  0 .

Level N | 95
Calculus I

dy
 x  y  1 .
2
54. Consider the difference equation given by
dx
a) Complete the following table

dy
Point dx
(1, 0)
(0, 0)
(1, 0)
(2, 1)
(1, 1)
(0, 1)
(1, 1)
(2, 1)

b) Sketch the slope field at the 8 points given in the table.

c) Use the slope field of the given differential equation to explain why a
solution could not have the graph shown below.
y

x
-2 -1 0 1 2

-1

d) Find the particular solution y  f  x  to the given differential equation

with the initial condition f  0    1 .

Level N | 96
Calculus I

dy
55. Given  x 4  y  1 .
dx
a) Sketch a slope field for the given differential equation at the twelve points
indicated.
y

  
3

  
2

  
1

 O  x
–1 1
b) The slope field is defined at every point in the xy-plane. Describe all points
in the xy-plane for which the slopes are negative.
c) Find the particular solution y  g  x  to the given differential equation
with the initial condition g  0  0 .

dy 1
56. Given:  x  y 2.
dx 3
a) Sketch a slope field for the given equation at the plotted nine points in the
coordinate plane.
y

 2 

 1 

 O  x
–1 1

d2y
b) In terms of x and y find .
dx 2

Level N | 97
Calculus I

c) Find the inequality that describes the region in the xy-plane for which all
d2y
 0.
dx 2
d) y  f  x  is a particular solution to the differential equation with the

initial condition f  0  2 . Does f  x  has a turning point at x = 0? What

is its nature?
e) Determine the values of the constants a and b, for which y  ax  b is a
solution to the differential equation.

57. Given the function by g  x   x3  2 x2  3x  2  2cos x , Let R be the shaded

region in the quadrant II bounded by the graph of g, the x-axis and y-axis. S is
the region bounded by the graph of g and line t, which is tangent to the graph of
g at x  1.5 .
a) Find the area of region of R.
b) Find the volume of the solid generated when R is rotated about y = –1.
c) Without actually evaluating, write an integral expression for the area of
region S.

(A graphic calculator is needed)


58. a) Find the area of the region R enclosed by the graph of y  3 x  1 , the
vertical line x  3 , and
the x-axis.
b) When R is revolved about the horizontal line y  3 , find the volume of the
solid generated.
c) When R is revolved about the vertical line x  7 , find the volume of the
solid generated.

Level N | 98
Calculus I

x3
59. Given that the region M is bounded by graphs of the curve y  , the line
2  x2
y = – x + 4 and the y-axis.
a) Sketch the given functions.
b) Find the area of M.
c) Find the volume of the solid generated when M is revolved about the line
y  0.
d) Find the volume of the solid whose base is the region M if the cross
sections taken perpendicular to the x-axis are equilateral triangles.

60. [A graphic calculator is needed to solve this question.]


Let S be the region in the first quadrant enclosed by the graph of

y  e2 x , y  2  2cos x
2
and the
y-axis.
a) Find the area of the region S
b) When region S is revolved about the x-axis a solid is generated, find the
volume of this solid.
c) The region S is the base of a solid. Consider each cross section, of this
solid, perpendicular to the horizontal axis as a rectangle whose height is
three times the width, where the width is bounded by the graphs of the
given functions. Find the volume of the solid.
61. Let R be the region in the first quadrant enclosed by the graphs of y  x and

y  x3 .
a) Find the area of R.
b) The region R is the base of a solid. For this solid, at each x the cross
 x 
section perpendicular to the x-axis has area A  x   cos   . Find the
 2 
volume of the solid.
c) Another solid has the same base R. For this solid, the cross sections
perpendicular to the y-axis are squares. Find the volume of this solid.

Level N | 99
Calculus I

62. For 0  t  3.5 , water flows into tank P at a rate of P  t   2t  4  4e3sin x gallons

per hour. In the same time interval, water flows into tank Q at a rate of Q  t  .

The graphs of P  t  and Q  t  intersect at t  m and t  2.800 . Both tanks are

initially empty.
a. Find the amount of water in tank P at t  3.5 .
b. During the time interval 0  t  m , water flows into tank Q at a constant
rate of 17.6 gallons per hour. Find the difference in the amount of water in
tanks P and Q at t = m.
c. The area of the region bounded by the graphs of P  t  and Q  t  for

m  t  2.800 is 74.833.
How much water is in tank Q at t  2.800 ?

63. a) Find the area of the region enclosed by y  3cos 2 x , 0  x  and the
4
axes.
b) The region in part (a) is rotated through 2 about the x-axis, find the
volume of the solid generated.

64. a) Find the area of the region enclosed by y  3cos 2 x , 0  x  and the
4
axes.
b) The region in part (a) is rotated through 2 about the x-axis, find the
volume of the solid generated.

Level N | 100
Calculus I

65. a) Sketch the graph of g  x   2 x  m


1 
b) Given f  x   ln  x  , x  0
2 
i) Sketch the graph of f  x 
ii) Find the equation of the normal to the graph of f  x  at x  2 .

66. a) Express the following as a single logarithm:


i) log5  log3
a a
ii) 5log2  3log4
b b
1
b) Express  9a 4  2 as an algebraic fraction its simplest form.

3 3  1 15  7 3
c) Show that  .
2 3 3 3

 x2  2
67. a) Show that 4ln x  ln(3x  2)  ln x 2  ln   , where x  .
 3x  2  3

b) Hence solve the equation 4ln x  ln(3x  2)  ln x 2  0 .

68.    
Solve the inequality log3 2 x 2  x  log 3 2 x 2  3x  1  1 .

7
8 x3
69. Find the exact value of  4 x2  9 dx
6
 3 
2
x
70. a) Show that x
0
2
5
dx  ln 
 5
.

b) Find x x  2 dx.

71. Solve the equation 25 x  15 giving the value of x correct to 3 significant


figures.
72. Use logarithms to solve the equation 22 x1  5 .

Level N | 101
Calculus I

73. Solve the equation 2 x  42 x1 .

 e  x  dx.
1
x
74. Find the exact value of
0

ex
75. a) i) Find  1  e x dx.
ex ln 3
ii) Hence evaluate 0 1  e x dx , giving your answer in the form ln
k, where k is an integer.
2
 ex 
b) i) Using the substitution u  1  e , find  
x
x 
dx.
1 e 
ii) Hence find the exact volume of the solid of revolution
ex
generated when the curve given by y  , between
1  ex
x   ln 3 and x  ln 3 , is rotated through 2 radians about the
x-axis.

Level N | 102
Calculus I

Top Questions for Chapter 8

76. Given: x and y are functions of time t that are related by the equation
y 2  3  xy .
dy dx
a) When t = 10, y= 3 and  8 . Find
dt dt
Given also that curve C: y 2  3  xy  0
b) Find the gradient of the curve.
c) Find the point(s), if they exist where the curve has a horizontal tangent.
d) Find the set of points which belong to C where the line tangent to the
1
curve is parallel to y  x.
3
77. The curve (C) has equation x  3cos y

3 π
a) Show that the point P  ,  lies on C
2 3
dy
b) Find at P.
dx
c) Find an equation of the normal to C at P.

78. Given f  x   e x 1  x

a) Show that f  x   0 has only one root and find this root to 4 decimal

places using the iterative formula xn1  e xn 1

b) Show that x  0.278 is a root for f  x   0 correct to 3 decimal places.

79. A curve has equation x2  xy  y 2  7  0 . Find the coordinates of the points


dy
on the curve where  0.
dx

80. A curve C is described by the equation 2 x2  3 y 2  x  3xy  1  0 . Find an

equation of the tangent and the normal to C at the point 1, 1 .

Level N | 103
Calculus I

Top Questions for Chapter 9

81. The line has vector equation r  8i  4 j  10k  λ  i  2j  3k  where λ is a

parameter. The point A has coordinates  7,6, m  where m is a constant. The

point B has coordinates  9, 2, n  where n is a constant. Points A and B lie on

line .
a) Find the values of m and n.
Point P lies on and such that OP is perpendicular to
b) Find the coordinates of P.
82. With respect to origin O the lines 1 and 2 are given by the equations

0  1
   
1 : r   3    2 
5 1
   
 5   q 
   
2 : r   13   t  1 , where  and t are parameters and p and q are
 10   2 
   
constants.
Given that 1 and 2 are perpendicular,

a) show that q  0

b) prove that A  5,14,8 is a point on 2. The point C lies on 1 and a

circle with center C cuts 2 at A and B.


c) find the position vector of B

Level N | 104
Calculus I

2
 
83. Relative to the origin O, the position vectors of points C and D are OC   3 
0
 
1
 
and OD   0  .
 4 
 
i) Find angle COD in degrees.
ii) Find the area of triangle COD giving your answer to two decimal
places.

iii) Find the unit vector in the direction of OC .


iv) Find the position vector for the midpoint M of CD.

v) Find the length of OM .


 m 
 
The position vector of point N is given by ON   1  .
 m  2
 
vi) If OM and ON have the equal lengths, find the value(s) of m.

84. Two planes, m and n, have equations 4 x  3z  5 and 4x  3 y  4 ,

respectively. Line l has equation r  i  j  k    3i  4 j

a) Show that l is parallel to n.


b) Find the position vector of the point of intersection of l and m.
c) A point P lying on l is such that its perpendicular distance from m and
n are equal. Find the position vectors of the two possible positions for
P and calculate the distance between them.
85. The points A and B have position vectors a and b relative to an origin O,
where a  5i  4 j  2k and b  7i  3 j  k .
a) Find the length of AB.
b) Use a scalar product to find angle OAB.

Level N | 105
Calculus I

 4  3 
86. Vectors u and v are given by u    and v    .
6  2
a) Find u  v and u  v .
b) Show that u  v  u  v .

87. Three points A, B and C have coordinates 1, 0, 7  , 13,9,1 and

 2, 1, 7  respectively.
a) Use a scalar product to find angle ACB.
b) Hence find the area of triangle ACB.
c) Show that a vector equation of the line AB is given by
r  i  7k  (4i  3j  2k) , where  is a scalar parameter.

 4 1 
   
88. The line 1 has vector equation r   7   λ 1  and the line 2 has vector
 4  3
   
 1 3
   
equation r   2   μ  9  where λ and μ are parameters.
 3 1 
   
The lines 1 and 2 intersect at the point B and the acute angle between them

is θ
a) Find the position vector of B
b) Find the value of cos  where  is the angle between the two lines.
The point A, which lies on 1 has position vector a  5i  8j  7k

The point C, which lies on 2 has position vector c  i  2j  3k


The point D is such that ABCD is a parallelogram.
c) Find the position vector of the point D.

Level N | 106
Calculus I

89. a) Find the value of m if the given pair of lines are intersecting
r  2i  3j  k    2i  7 j  k  and r  i  3j  t  mi  6j  k 

b) Referred to a fixed origin, the points A and B have position vectors


 i  2j  3k  and  5i  3j respectively. Find, in vector form, an

equation of the line 1 which passes through A and B.

90. a) Find an equation of the line 1, passing through the two points

A  1,1,1 and B  2,3,5 .

 1  2 
   
b) The line 2 of equation r   2   t  1  intersects 1 at point C, find
 3  2
   
the position vector of C.
c) Point M 1,3,5 is on 2. Find the shortest distance from M to 1.

 1   3  5
     
91. Show that the two lines r   1   t  5  and r    6  do not intersect and
 0   2  0
     
find the shortest distance between them.

 1   2  0  3
       
92. The lines 1 and 2 are of equations r   1   t  1  & r   2     1 
 3   3  0  3 
       
respectively, where  and t are scalar parameters.
a) Find the position vector of the point C the intersection of 1 and 2.

b) If A is a position on 1 corresponding to t  1 and B is a point on 2

corresponding to   1 , calculate the area of triangle ABC to the nearest whole


number.

Level N | 107
Calculus I

0 0
   
93. With respect to a fixed origin O, the line l has equation r   1     1 ,  is
 2 1
   
a scalar parameter.
a) Show that the point N  0, 1, 4 lies on line l.
2p
 
The point M has position vector  0  relative to O, where p is a constant and
p
 

MN is perpendicular to l,
b) find the value of p
Given that B is a point on l such that BMN  45 ,
c) find the coordinates of the two possible positions of B.
1  1   1  4 
       
94. The two lines 1 : r   2   t  5  and 2 : r   2   λ 10  intersect at point
5  0  3  2 
       
A
a) Find the position vector of point A
b) Find the acute angle between the two lines
c) Show that B  0, 3,5 belongs to 1 and C  9, 18, 1 belongs to

2.

d) Find the exact value of the area of ABC .

95. Two intersecting straight lines have equations


x  5 y  11 z  7 x 9 y 4 z  4
  and   .
4 3 5 2 1 4
Find the coordinates of their point of intersection.

Level N | 108
Calculus I

96. The points A and B have position vectors i  j  k and 2i  j  3k


respectively, relative to the O. The point C is the line OA extended so that

AC  2OA and the point D is on the line OB extended so that BD  3OB . The
point X is such that OCXD is s parallelogram.
a) Show that a vector equation of the line AX is
r  i  j  k  (5i  j  7k) and find an equation of the line CD in a
similar form.
b) Prove that the line AX and CD intersect and find the position vector of
their point of intersection.

Level N | 109
Calculus I

Top Questions for Chapter 10

m
97. The complex number z is given by z  where m . Given that z  12 ,
3  2i
find z in the form a  ib .
 5π 5π 
98. Given that z  2  cos  i sin  and w  2  2i , find
 6 6 
z z
a) and arg   in radians as a multiple of π .
w  w
b) On an argand diagram plot points A, B, C, D representing z,
z
w, and zw.
w
c) Find AOB and the area of AOC

99. a) If z =  cos  i sin   , find and simplify an expression for z n  z  n .


3
 1
b) Expand  z   and use your expansion to find an expression for
 z
sin  .
3

c) By using a similar procedure, obtain an expression for cos4  .


d) Use De moivre’s theorem to find an expression for sin 4 .Hence,
solve the equation sin 4  8sin  cos2   0 .

100. a) Solve z 2  2 z  2  0 , giving your answer in the form x  iy, x, y 


b) Find the modulus and argument of each root.
z3
c) If z1 and z 2 are roots of the equation given in (a), write in the form
a  ib, a, b 

Level N | 110
Calculus I

101. The complex number z is given by z  3  3 i


a) Find the modulus and argument of z.
b) The complex conjugate of z is denoted by z*. Showing your working,
express the following in the form x  iy , where x and y are real.
i) 3z  2 z 
iz 
ii)
z
c) On a sketch of an Argand diagram with origin O, show the points M
and N representing the complex number z and iz*, respectively. Prove

that angle MON  .
6
102. The complex number 4i is denoted by u . The complex number with modulus
2
1 and argument  is denoted by w .
3
a) Find in the form x  iy , where x and y are real, the complex numbers
u
w, uw, and .
w
b) Sketch an Argand diagram showing the points K, L and M representing
u
the complex numbers u, uw, and , respectively.
w
c) Prove that triangle KLM is equilateral.

Level N | 111
Calculus I

103. The complex number 3  i is denoted by u . Its complex conjugate is denoted


by u*.
a) Show, on a sketch of an Argand diagram with origin O, the points K, L
and M representing the complex numbers u, u* and u + u*,
respectively. Describe in geometrical terms the relationship between
the four points O, K, L and M.
u
b) Express in the form x  iy , where x and y are real.
u
u
c) Considering the argument of , or otherwise, prove that
u
3 1
tan 1    2 tan 1   .
4  3
104. The complex number 3  4i is denoted by z .Giving your answers in the
form x  iy and showing clearly how you obtain them, find

a) 2z  z 
5
b)
z
c) Show z and z  on an Argand Diagram.

105. The complex number z is given by 20  21i .Showing all your working.
a) Find the value of z .

b) Calculate the value of arg z correct to 3 significant figures.


c) Express 1/ z in the form x  iy where x and y are real numbers.

2
106. a) The complex number is such that z  2 and arg z   .
3
Find the exact value of the real
part of z and the imaginary part of z .
b) The complex numbers u and v are such that u  1  ia and
v  b  i where a and b are real and a  b . Given that uv  7  9i find
the values of a and b.

Level N | 112
Calculus I

1 1
107. It is given that y   , where x and y are real positive, and i 2  1 .
x i x i

1 1 y2
a) Show that x  and y  1.
y
b) Deduce that xy  2.
c) Indicate the region in the xy-plane defined by y  1 and xy  2 .

108. a. Solve the equation z 3  1  0 expressing your answers in the form


z  rei .
b. Solve the equation z3  4z 2  6z  4  0 and show that the solutions are
the vertices of an isosceles right triangle.
109. Sketch the following in an argand diagram.
1
a)    arg z  0
2
b) z 3  5

c) z 2  z 3

110. The point A represent the complex number z in an Argand diagram.


Given that z  2  i  3

a) Sketch the locus of P in an Argand diagram.


b) Find the exact values of the maximum and minimum of z

111. a) Solve the equation z 2  4iz 13  0 , giving your answer in the form
x  iy ,
where x and y are real.
b) Find the modulus and argument of each root.
c) Sketch and Argand diagram showing the points representing the roots.

Level N | 113
Calculus I

u
1  2i 
2
112. The complex number u is defined by .
4  3i
a) Without using a calculator and showing your working, express u in
the form x  iy , where x and y are real.
b) Sketch an Argand diagram showing the locus of the complex number z
such that z  u  u .

5  3i
113. The complex number u is given by u  .
4i
a) Express u in the form x  iy , where x and y are real.
b) Find the modulus and argument of u .
c) Sketch an Argand diagram showing the point representing the complex
number u . Show on the same diagram the locus of the point
representing the complex number z such that z  u  1 .

d) Using your diagram, calculate the least value of z for points on this

locus.
114. A root of the equation z 2  pz  q  0 is 3  i , where p and q are real. Write
down the other root of the equation and hence calculate the values of p and q.

115. a)   
Express z 4  3z 2  4 in the form z 2  a z 2  b where a and b are real

constants to be found.
b) Hence draw an Argand diagram showing the points that represent the
roots of the equation z 4  3z 2  4 .

116. a) Verify that z  1 is a root of the equation z3  5z 2  9z  5  0.


b) Find the two complex roots of the equation z3  5z 2  9z  5  0.
c) Show all three roots on an Argand diagram.

Level N | 114
Calculus I

117. The roots of the equation ,𝑧-2..−6z + 10 = 0 are z1 = 3 + i and z2 = 3 - i


a) Write down the value of z 2 .

b) Show z1 and z 2 on an Argand diagram

c) Show that z12  8  6i .

118. a) Solve the equation  2  i  z   4  in  . Give your answer in the form

of a  ib expressing
a and b in terms of the real constant n.
b) The roots of the equation z 2  8z  25  0 are denoted by z1 and z2 .

i) Find z1 and z2 and show these roots on an Argand diagram.


ii) Find the modulus and argument in radians of each of

 z1  1 and  z2  1 .
119. a) The equation 3x3  2 x2  2 x  8  0 has one real root and two complex
roots.
Showing your working, verify that 1  i is one of the complex roots.
b) On a sketch of an Argand diagram, show the point representing the
complex number 1  i . On the same diagram, shade the region whose
points represent the complex numbers z which satisfy both the
1
inequalities z  1  i  1 and arg z   .
4

Level N | 115

You might also like